Poverty & Race

PRRAC POVERTY & RACE RESEARCH ACTION COUNCIL

September/October 2004 Volume 13: Number 5

Schools and the Achievement Gap: A Symposium

Almost everyone, Right as well as Left,recognizes the great disparities that currently exist between the education generally received by poor and minority students compared to that received by white middle- and upper-class students. Richard Rothstein, in his new book, Class and Schools: Using Social, Economic, and Educational Reform to Close the Black-White Achievement Gap (Teachers College/Columbia Univ. & Economic Policy Inst., 2004, 203 pp.), makes a powerful case that the income/wealth, residential, employment and other powerful disparities that characterize our society are responsible for and perpetuate these educational disparities — which in turn reinforce and perpetuate these other, larger disparities. His book makes a strong case for major economic and social reform, absent which reform in school policy and programs can have only limited benefit to those the education system now is failing. We asked him to prepare a summary of his argument, then asked a range of commentators, Left and Right, to respond to his argument, with Rothstein’s response to those eight comments closing out the symposium. We’d be happy to hear from readers and may publish further comments and letters in the next issue of P&R. PRRAC law student intern Nicole Devero assisted in formulating and overseeing the symposium - CH

Even the Best Schools Can’t CONTENTS: Close the Race Achievement Gap Schools and the Achievement Gap: by Richard Rothstein A Symposium What Schools The achievement gap between tions overlap. While average achieve- Can’t Do ...... 1 poor and middle-class, black and white ment of low-income students is below Commentaries by: children is an educational challenge, average achievement of middle-class Pedro A. Noguera ..... 11 but we prevent ourselves from solv- students, there are always some John H. Jackson ...... 12 ing it because of a commonplace be- middle-class students who achieve be- Jenice L. View ...... 13 lief that poverty and race can’t “cause” low typical low-income levels. Some Stan Karp ...... 14 low achievement and that therefore low-income students achieve above Wendy Puriefoy ...... 14 schools must be failing to teach disad- typical middle-class levels. “Demog- Mark Simon ...... 16 vantaged children adequately. After raphy is not destiny,” but students’ Krista Kafer ...... 16 all, we see many highly successful stu- family characteristics are a powerful Dianne M. Piché/ dents from lower-class backgrounds. influence on their relative average Tamar Ruth ...... 17 Their success seems to prove that so- achievement, even in the best of Response ...... 20 cial class cannot be what impedes most schools. What Works ...... 3 disadvantaged students. Widely repeated accounts of schools HHS and the Health Yet the success of some lower-class that somehow elicit consistently high Care Gap ...... 4 students proves nothing about schools’ achievement from lower-class children Regional Organizing ... 5 power to close the achievement gap. almost always turn out, upon exami- PRRAC Update ...... 15 There is a distribution of achievement nation, to be flawed. In some cases, Resources ...... 22 in every social group. These distribu- (Please turn to page 2)

Poverty & Race Research Action Council • 3000 Connecticut Avenue NW • Suite 200 • Washington, DC 20008 202/387-9887 • FAX: 202/387-0764 • E-mail: [email protected] • www.prrac.org Recycled Paper (GAP: Continued from page 1) some poor children proves that social where landlords use high-sulfur home disadvantage does not cause low heating oil, and where diesel trucks “schools that beat the odds” are highly achievement. frequently pass en route to industrial selective, enrolling only the most able and commercial sites, such children are or most motivated lower-class chil- more likely to suffer from asthma, dren. Some are not truly lower-class Social Class leading to more absences from school schools – for example, schools enroll- and Learning and drowsiness (from lying awake ing children who qualify for subsidized wheezing at night) when present. Re- lunches because their parents are Partly, our confusion stems from cent surveys of black children in Chi- poorly paid but highly educated. Some failing to examine the concrete ways cago and in City’s Harlem schools “succeed” with lower-class that social class actually affects learn- community found one of every four children by defining high achievement ing. Describing these may help to make children suffering from asthma, a rate at such a low level that all students can their influence more obvious. six times as great as that for all chil- reach it, despite big gaps that remain Overall, lower-income children are dren. Asthma is now the single big- at higher levels. And some schools’ in poorer health, and poor health de- gest cause of chronic school absence. successes are statistical flukes – their presses student achievement, no mat- Because primary care physicians are high test scores last for only one year, ter how effective a school may be. few in low-income communities (the in only one grade and in only one sub- physician to population ratio is less ject. than a third the rate in middle-class While the idea that “if some chil- Partly, our confusion communities), disadvantaged children dren can defy the demographic odds, stems from failing to (even those with health insurance) are all can” seems plausible, it reflects a examine the concrete also more likely to miss school for rela- reasoning whose naiveté we easily rec- tively minor problems, like common ognize in other policy areas. In hu- ways that social class ear infections, for which middle-class man affairs, where multiple causation actually affects children are treated promptly. If in is typical, causes are not disproved by learning. attendance, children with earaches exceptions. Tobacco firms once have more difficulty paying attention. claimed that smoking does not cause Each of these well-documented so- cancer because we all know people Low-income children have poorer vi- cial class differences in health is likely who smoked without getting cancer. sion, partly because of prenatal con- to have a palpable effect on academic We now consider such reasoning spe- ditions, partly because, even as tod- achievement. The influence of each cious. We understand that because no dlers, they watch too much television may be small, but combined, the in- single cause is rigidly deterministic, both at home and in low-quality fluence of all is probably huge. some people can smoke without harm, daycare settings, so their eyes are more The growing unaffordability of ad- but we also understand that, on aver- poorly trained. Trying to read, their equate housing for low-income fami- age, smoking is dangerous. Yet despite eyes may wander or have difficulty lies also affects achievement – children such understanding, quite sophisticated tracking print or focusing. A good part whose families have difficulty finding people often proclaim that success of of the over-identification of learning stable housing are more likely to be disabilities for lower-class children is mobile, and student mobility is an probably attributable simply to undi- important cause of failing student per- Poverty and Race (ISSN 1075-3591) is published six times a year by the agnosed vision problems for which formance. [See “High Classroom Poverty & Race Research Action therapy is available and for which spe- Turnover: How Children Get Left Council, 3000 Conn. Ave. NW, #200, cial education placement should be un- Behind, Poverty & Race, May/June Washington, DC 20008, 202/387- necessary. 2002.] A 1994 government report 9887, fax: 202/387-0764, E-mail: Lower-class children have poorer found that 30% of the poorest chil- [email protected]. Chester Hartman, oral hygiene, more lead poisoning, dren had attended at least three differ- Editor. Subscriptions are $25/year, more asthma, poorer nutrition, less ent schools by third grade, while only $45/two years. Foreign postage ex- tra. Articles, article suggestions, let- adequate pediatric care, more exposure 10% of middle-class children did so. ters and general comments are wel- to smoke, and a host of other health Blacks were more than twice as likely come, as are notices of publications, problems - on average. Because, for as whites to change schools this much. conferences, job openings, etc. for our example, lower-class children typically It is hard to imagine how teachers, no Resources Section. Articles generally have less adequate dental care, they are matter how well trained, can be as ef- may be reprinted, providing PRRAC more likely to have toothaches and fective for children who move in and gives advance permission. resulting discomfort that affects con- out of their classrooms. © Copyright 2004 by the Poverty & Race Research Action Council. All centration. Differences in wealth are also likely rights reserved. Because low-income children are to affect achievement, but these are more likely to live in communities (Please turn to page 7)

2 • Poverty & Race • Vol. 13, No. 5 • September/October 2004 What Works: A Fifty Year Retrospective by David Barton Smith Three watershed events in the ible, however, rarely forces change. 4. Regionalism: Patterns of geo- struggle to end divisions by race in the There are just too many more com- graphic and residential segregation are marking major an- fortable, moralistic and victim-blam- limit the ability to reduce unequal niversaries: the 1954 Brown v. Board ing alternative explanations. Random- treatment. Treatment may be inte- of Education decision, the Civil Rights ized testing varying only the race of grated and equal within school districts Act of 1964 and the 1965 Medicare- the testers clears away this ideological or health systems but unequal between. Medicaid legislation. While we have underbrush. By the 1970s, testing was The more affluent and predominantly fallen far short of the vision of the being used by fair housing agencies to white suburban areas do better. Health movement that produced these events, determine the validity of housing dis- systems and school districts that don’t what has worked? I list concrete ex- crimination complaints. In 1979, the overlap such boundaries can do little amples of five general strategies that Department of Housing and Urban to reduce the overall level of segrega- have given good returns on invest- Development sponsored the first na- tion and are limited in their ability to ments. tional testing study of discrimination address treatment disparities. For a 1. Visibility: Nothing happens un- in housing markets. The study dem- brief period in the 1970s and 1980s, til the inequities and disparities are onstrated the feasibility of such sur- federal regional health planning cer- made visible. The Medicare-Medicaid veys and the persistence of a high de- tificate of need requirements forced legislation was passed only after dis- integration of specialized health ser- parities in access to care by race and Choice was viewed as vices in many metropolitan areas. income began to be documented by Metropolitan areas that had been op- regular national surveys. Since the the wolf in sheep’s erating under city-suburban court-or- 1989 revisions of the Home Mortgage clothing that would dered desegregation have achieved a Disclosure Act of 1975 (HMDA), undermine the goal of greater degree of integration. In gen- residential mortgage lenders are re- integration. eral, metropolitan areas whose schools quired to publicly report detailed in- or health systems are regionalized have formation, including the race of loan fewer disparities and better overall out- applicants. Nationally, loan approval gree of discrimination in the housing comes. rates, unadjusted for risk, are substan- market. This has been followed by a 5. Universality: “Freedom of tially lower for blacks than whites. series of regular testing studies that choice” was the rallying cry of the seg- The Federal Reserve Bank of Boston have kept pressure on and have docu- regationists in the 1960s and is em- in 1992 did the first “risk adjusted” mented progress in reducing the level bedded within market/competitive so- study. The study concluded that mi- of discrimination. Perhaps reflecting lutions to schools and healthcare. The norities in the Boston Area were re- these pressures,the Census documents initial success of the Medicare pro- jected for loans 56% more often than a modest decline in residential segre- gram in integrating hospitals was based equally creditworthy whites. After gation in most metropolitan areas over on a single universal program (all per- scathing headlines, heated industry re- the last 20 years. sons over 65) and a restructuring of buttals and lending agency efforts to 3. Gold: The golden rule in the hospital system to restrict consumer improve the fairness of their loan ap- America is that those that have the gold choice. This meant one entrance, one plication processes, the number of rule. Title VI of the Civil Rights Act waiting room and race-blind room as- loans approved nationally for blacks attempted to impose the condition of signment. The goals of desegregation has increased and rejection rates have integration and non-discrimination on and equity trumped individual con- declined. Public disclosure reports by all organizations receiving federal sumer choice. Choice was viewed as race for individual lending institutions funds. Unwavering commitment to the wolf in sheep’s clothing that would are available both in hard copy and this principle in implementation of the undermine the goal of integration. from the web site of the Federal Fi- Medicare program worked. Almost These five general strategies have nancial Institutions Examination 1,000 hospitals integrated their accom- worked because the majority of Ameri- Council (www.ffiec.gov). As a re- modations and medical staff in a pe- cans believe (or at least can be shamed sult, lenders concerned about their pub- riod of a few months. The visible sym- into saying they believe) in equal op- lic image have a strong incentive to bols of Jim Crow in the nation’s hos- portunity and that segregation and dis- demonstrate good faith by following pitals disappeared almost overnight, crimination should not be tolerated. such best practice loan fairness guide- and gross racial disparities in access to Yet, the sheep’s clothing arguments of lines. services gradually disappeared over the the wolf of segregation have blunted 2. Testing: Making disparities vis- next decade. (Please turn to page 6)

September/October 2004 • Poverty & Race • Vol.13, No. 5 • 3 Why Is HHS Obscuring a Health Care Gap? by H. Jack Geiger

Over the past four years my col- volved. Much needs to be learned. But times receive.” Worse, the new sum- leagues and I have read and reviewed even though there are at least eight mary begins with a short list of rela- more than a thousand careful, peer- major reviews of all this evidence, in- tively minor health areas in which reviewed studies documenting system- cluding the Institute of Medicine’s minority and poor populations do atic deficiencies and inequities in the landmark 2001 study, “Unequal Treat- slightly better than the majority (be- health care provided for African ment,” there has been no overall na- cause, an AHRQ spokesman said, Americans, Hispanics, Native Ameri- tional assessment of the scope of the “Secretary [Tommy] Thompson likes cans and members of some Asian sub- problem. So it was a welcome devel- to focus on the positive.”) groups. The evidence is overwhelm- opment when Congress mandated There is a pattern here. A few weeks ing. Unfortunately, the Department of HHS’s Agency for Healthcare Re- ago, all of the different institutes that Health and Human Services seems in- search and Quality (AHRQ), a body make up the National Institutes of tent on papering it over. with an impeccable track record of Health released their draft “strategic This is the only conclusion that can expertise and honesty, to produce the plans” to overcome racial and ethnic be drawn from HHS’s recent treatment first annual national report card on disparities in health status — the bur- of the first national report card on dis- disparities. dens of greater illness and shorter life parities in the diagnosis and treatment expectancies of America’s minority for this country’s most vulnerable “Of all the forms of populations. Disparities in health care populations. The department edited obviously contribute to those burdens. and rewrote the report’s summary un- injustice, discrimination But only three of the NIH’s 14 insti- til it reflected nothing close to reality. in health care is the tutes even mentioned them. The reality is this: If you are an Af- most cruel.” A recent report by a panel of ex- rican American man with one form of perts convened by Physicians for Hu- lung cancer, you are far less likely than man Rights recommended corrective a similarly ill white patient to receive The AHRQ did its job well. Its draft steps to be taken by government at a surgical procedure that would cut report was a clear and massive presen- every level, as well as by the medical your chances of early death nearly in tation of the data on disparities in care profession, hospitals, HMOs, commu- half, from 95 percent to 50 percent — associated with race, ethnicity and so- nity groups and civil rights organiza- even if you have the same health in- cioeconomic status. Its summary was tions. But the federal government has surance coverage and are in the same blunt, noting that such disparities are an especially critical role to play in col- hospital. If you are a Hispanic trauma “national problems that affect health lecting and honestly analyzing data, victim with multiple bone fractures, care at all points in the process, at all supporting a more diverse health you are less likely to be given adequate sites of care, and for all medical con- workforce, and ensuring enforcement pain medication — or any at all. If you ditions,” affecting health outcomes and of civil rights in the health care sys- are a low-income or minority child entailing “a personal and societal tem. To avoid the truth, or cloak it in with severe asthma, your chances of price.” more comfortable words, is to aban- getting the most effective drug com- After “review” by HHS, those don that responsibility. binations are slimmer, and you endure truthful words are gone, as are most The Rev. Martin Luther King Jr. repeated attacks of the disease and hos- references to race and ethnicity, now understood what is at issue here. “Of pitalizations. Native Americans with described as problems that existed “in all the forms of injustice,” he said, diabetes, or Asian/Pacific Islanders the past.” Prejudice is “not implied in “discrimination in health care is the with HIV-AIDS, all too often experi- any way.” Disparities are simply called most cruel.” ence such disparities in care. The pat- “differences,” and — incredibly — tern extends over the full range of ”there is no implication that these dif- H. Jack Geiger ([email protected]) medical conditions. ferences result in adverse health out- is Arthur C. Logan professor emeritus The reasons are complex. Patients comes.” of community medicine at the City often mistrust the medical system be- What of the thousand or more stud- University of New York Medical School cause of perceived past discrimination. ies to the contrary? The new summary and a past president of Physicians for On the physicians’ side, poor commu- says: “Some studies and commenta- Human Rights. This article first ap- nication, lack of cultural understand- tors have suggested that a gap exists peared in the Jan. 27, 2004 Washing- ing, and subconscious negative racial between ideal health care and the ac- ton Post and is reprinted with the and ethnic stereotyping can be in- tual health care that Americans some- author’s permission. ❏

4 • Poverty & Race • Vol. 13, No. 5 • September/October 2004 Organizing for Regional Equity: The Gamaliel Foundation by Jill Mazullo

Where can you find the crux of became familiar with the work of ur- By re-framing their arguments from the Civil Rights Movement today? ban affairs specialists like David Rusk, a regional perspective, these individu- Some say it’s emerging in the Gamaliel Myron Orfield and john powell, all als move from a powerless position Foundation and the work of its affili- of whom conduct research in the areas where they are talking amongst them- ate organizations, such as MICAH (the of social and fiscal equity, land use selves into a diverse coalition of Milwaukee Inner-City Congregations reform and regional governance. churches, synagogues, neighborhood Allied for Hope), the Jubilee Interfaith Gamaliel staff members quickly saw organizations, leagues of cities and Organization in New Jersey, or ways that the academics’ arguments for other like-minded organizations oper- MOSES (Metropolitan Organizing regionalism could be applied to their ating under one banner. Together, the Strategy for Enabling Strength) in community organizing model. coalition can lend a moral component Detroit. They are just 3 of the 55 such As a result, Gamaliel asked Rusk, to many social policy arguments, such grassroots, multi-racial organizations Orfield, powell and George Ranney to as demanding affordable housing in that operate as affiliates of the serve as ongoing strategic partners. affluent suburbs, better bus service for Gamaliel Foundation, an organizing Gamaliel taps these regional affairs low-income neighborhoods, and ad- institute headquartered in Chicago. equate school funding so children When executive director Greg The coalition can lend a throughout the region have access to a Galluzzo came on board in 1986, he decent education. charged the foundation with training moral component to The new organization then hires a new leaders in the style of Saul many social policy strong community organizer who can Alinsky, the famed father of commu- arguments. train people to conduct surveys of the nity organizing who emerged a leader coalition members. Ultimately there in the tough stockyard neighborhoods may be up to 2,000 surveys collected of Chicago. While Alinsky had theoreticians and academics and puts asking for core issues the individuals worked on a neighborhood scale, em- their analyses into the hands of orga- want to see addressed. The results are powering residents and workers to de- nizers who can effectively move from boiled down into the three issue areas mand social reforms such as better theory to practice. for reform that emerge from the sur- housing, safer working conditions and The Gamaliel model generally vey. The organizers work with lead- lower crime, the Foundation has works in the following fashion: A ership at Gamaliel to draft an agenda broadened its organizational emphasis handful of concerned grassroots activ- for reform on the three emerging is- to a regional scale—lately even tack- ists from anywhere in the U.S. will sue areas. The organizers then host ling policy issues that are national in contact the Foundation to discuss a re- large meetings with upwards of 1,500 breadth. gional problem they’re facing. The people, where strategic elected offi- Following the lead of Minnesota or- issue may be school funding, or cials are in attendance. The organiz- ganizer Pamela Twiss, Gamaliel staff sprawling development, or providing ers and the meeting attendees aim to adequate mass transportation to low- hold elected officials accountable by income residents. The citizens may asking them to commit publicly to The source of the Gamaliel (pro- have been meeting independently for voting favorably on regional bills nounced gah-MAY-lee-ehl) Foun- months, even years, and seek assis- forthcoming in their legislature. dation name: Gamaliel is a Bibli- tance in building a broader coalition. This model has worked many times cal figure from the New Testament With Gamaliel’s help, the small band in metropolitan regions throughout the who served as a mentor to Paul, of activists learns that the issues con- U.S. and produced successful legisla- the patron saint of organizers. The cerning them are likewise of impor- tion embraced by the Gamaliel coali- Gamaliel Foundation web site: tance to churches throughout the re- tions. The affiliates have organized www.gamaliel.org. Link to gion, in the core city as well as older, successful, racially integrated coali- download “An Activist’s Guide to low tax base suburbs. Even organiza- tions for broad reforms on land use, Metropolitics: Building Coalitions tions in property-wealthy suburbs have transportation, fair housing, tax eq- for Reform in America’s Metro- self-interest in joining such a regional uity, school funding reform, health politan Areas,” by Myron Orfield: coalition; their faith requires that they care and immigrant rights. Here are www.ameregis.com serve those with little means to pro- just a handful of reforms they’ve tect themselves. (Please turn to page 6)

September/October 2004 • Poverty & Race • Vol.13, No. 5 • 5 (EQUITY: Continued from page 5) dump in a rural location removed Gamaliel is very intentional in its pro- from densely populated urban ar- motion of people of color, working helped achieve: eas was a better approach. The key hard to ensure they have prominent for IF was repositioning the issue leaders of all races. Galluzzo says it’s 1) Passing a fair housing bill: Min- from a single-minded focus on important that newcomers to the orga- nesota state legislator Myron economic development to the nization see diverse leadership so they Orfield was determined to pass a moral issue of who would have to can envision themselves moving up the fair housing bill in the early 1990s live with the nuisance of the waste ranks into key roles in the future. to take the pressure off the central dump itself. IF was successful in Gamaliel leaders have challenged cities to house all of the region’s framing the issue, and the waste themselves to grow in response to shift- low-income population. He ar- dump was ultimately built in a ing policies in the U.S. Their current gued that suburbs ought to pro- rural locale. push is a national campaign for civil vide a portion of affordable hous- rights for immigrants called Rolling ing based on the regional need. Thunder, with dozens of large meet- Orfield pursued the bill three years In many ways, Gamaliel ings planned for this fall across the in a row. The first year, the tenor is the Civil Rights country. Galluzzo says it’s a difficult of the debate was just plain ugly: Movement of today. path since the organization is currently Legislators from wealthy suburbs better tooled for regional reforms, but could hardly conceal their disdain the leadership clearly stated they need for the low-income people who 3) Creating a regional transit au- to defend the rights of immigrants who might relocate to their commu- thority: Transit in the Detroit area cannot speak for themselves out of fear nity. But the second year, a num- has been conducted piecemeal for of deportation. ber of faith-based organizations decades, lacking coordination In many ways, Gamaliel is the Civil associated with Gamaliel spurred from one county to the next. Rights Movement of today. They can priests and pastors to turn out at Transportation activists in Michi- rally a crowd as few can, and are mul- legislative hearings. The pastors gan thought this was inefficient, tiracial in thought and deed. Watch also paid personal visits to outspo- since mass transit is regional by what they can do in your region. ken legislators, challenging them nature and crosses many jurisdic- to defend their basest comments. tional lines. MOSES pressured Jill Mazullo ([email protected]) is The presence of men and women local and state politicians to think a research fellow at the Institute on in pastoral garb brought out the beyond their borders and consider Race & Poverty at the University of best in the legislators, and the de- coordinating the transit systems. Minnesota Law School, which is di- bate became more substantive, and With Gov. Jennifer Granholm on rected by Myron Orfield. ❏ more polite as well. The third their side, the three-county Detroit year, a housing compromise Area Regional Transportation passed, allowing the seven-county Authority was founded. This and (WHAT WORKS: Continued from page 3) Metropolitan Council to negoti- other organizing successes led ate housing goals and withhold re- Gamaliel to name MOSES the or- the effectiveness of each of these strat- gional services to cities that did ganization of the year in 2002. egies. Visibility has been fought with not participate in the regional privacy objections, testing by raising housing fund. These policy wins and many more the specter of costly government in- by Gamaliel affiliates across the coun- trusion, regionalization by the rheto- 2) Siting a landfill: A landfill was try are a real testament to the organiz- ric of community empowerment and originally proposed to be sited ing model at the heart of the institute. entrepreneurship, and universality by within the city limits of Gary, In- The coalitions bring people together appeals for consumer choice and com- diana. Initially, even elected of- around seemingly intractable policy petition. The vision of the Civil Rights ficials in Gary wanted the waste issues. The organizers choose their is- Movement will be realized to the ex- dump to be located in their city sues carefully, zeroing in on winnable tent that the wolf is named for what it for the sake of economic devel- battles for which they can provide is and the long-term impact of such opment. But the Interfaith Federa- workable solutions their members agree alternatives on the cost and quality of tion (IF), a Gamaliel affiliate, upon. life for all citizens made clear. said the proposed site was irre- Gamaliel is notable in its ability to sponsible because the waste dump attract and retain people from all faiths, David Barton Smith (dbsmith2@ would be situated in a low-in- all races, all classes; the affiliate orga- comcast.net) is Professor, Department come, largely black neighbor- nizations consist of workers, students, of Risk, Insurance and Healthcare hood. IF argued that placing the ministers, laypeople and more. Management at Temple University. ❏

6 • Poverty & Race • Vol. 13, No. 5 • September/October 2004 (GAP: Continued from page 2) ploma or less benefit from daily read- Great Britain, and as do Okinawans ing. White children are more likely and low-caste Buraku in Japan. usually overlooked because most ana- than blacks to be read to in pre-kin- An international survey of 15-year- lysts focus only on annual family in- dergarten years. olds, conducted in 2000, found a come to indicate disadvantage. This A five-year-old who enters school strong relationship in almost every makes it hard to understand why black recognizing some words and who has nation between parental occupation and students, on average, score lower than turned pages of many stories will be student literacy. The gap between lit- whites whose family incomes are the easier to teach than one who has rarely eracy of children of the highest status same. It is easier to understand this held a book. The latter can be taught, workers (like doctors, professors, law- pattern when we recognize that chil- but the child with a stronger home lit- yers) and the lowest status workers dren can have similar family incomes eracy background will typically post (like waiters and waitresses, taxi driv- but be of different economic classes: higher scores on reading tests than one ers, mechanics) was even greater in black families with low income in any for whom book reading is unfamiliar Germany and in the United Kingdom year are likely to have been poor for — even if both children benefit from than it was in the United States. After longer than white families with simi- high expectations and effective teach- reviewing these results, a U.S. Depart- lar income in that year. White fami- ing. So, the achievement gap begins. ment of Education summary concluded lies are likely to own far more assets that “most participating countries do that support their children’s achieve- not differ significantly from the United ment than are black families at the same Homework exacerbates States in terms of the strength of the income level, partly because black academic differences relationship between socioeconomic middle-class parents are more likely between middle- and status and literacy in any subject.” Re- to be the first generation in their fami- working-class children markably, the Department published lies to have middle-class status. Al- because middle-class this conclusion at the very time it was though median black family income guiding a bill through Congress — “No is now nearly 2/3 of white income, parents are more likely Child Left Behind” — that demanded black family assets are still only 12% to assist with every school in the nation abolish so- of whites’. This difference means that, homework. cial class differences in achievement among white and black families with within 12 years. the same middle-class incomes, the Urging less educated parents to read whites are more likely to have savings If a society with such differences to children can’t fully compensate for for college. This makes white wants children, irrespective of social differences in school readiness. If chil- children’s college aspirations more class, to have the same chance to dren see parents read to solve their own practical, and therefore more com- achieve academic goals, it should find problems or for entertainment, chil- monplace. ways to help lower-class children en- dren are more likely to want to read ter school having the same familiarity themselves. Parents who bring read- with books as middle-class children ing material home from work demon- Child Rearing/ have. This requires re-thinking the strate by example to children that read- Personality Traits institutional settings in which we pro- ing is not a segmented burden but a vide early childhood care, beginning seamless activity that bridges work and Social class differences however, in infancy. leisure. Parents who read to children amount to more than these quantifi- Some people acknowledge the im- but don’t read for themselves send a able differences in health, housing, pact of such differences but find it hard different message. income and assets. There are power- to accept that good schools should How parents read to children is as ful social class differences in child rear- have so difficult a time overcoming important as whether they do; more ing habits and personality traits, and them. This would be easier to under- educated parents read aloud differ- these too cause average differences in stand if Americans had a broader in- ently. When working-class parents read academic achievement by social class. ternational perspective on education. aloud, they are more likely to tell chil- Consider how parents of different Class backgrounds influence relative dren to pay attention without interrup- social classes tend to raise children. achievement everywhere. The inabil- tions or to sound out words or name Young children of more educated par- ity of schools to overcome the disad- letters. When they ask children about ents are read to more consistently, and vantage of less literate homes is not a a story, questions are more likely to are encouraged to read more by them- peculiar American failure but a uni- be factual, asking for names of objects selves when they are older. Most chil- versal reality. Turkish immigrant stu- or memory of events. dren whose parents have college de- dents suffer from an achievement gap Parents who are more literate are grees are read to daily before they be- in Germany, as do Algerians in more likely to ask questions that are gin kindergarten; few children whose France, as do Caribbean, African, creative, interpretive or connective, parents have only a high school di- Pakistani and Bangladeshi pupils in (Please turn to page 8)

September/October 2004 • Poverty & Race • Vol.13, No. 5 • 7 (GAP: Continued from page 7) behind an order and to internalize it. personality traits at home when they Middle-class parents implicitly begin design activities where children figure like “what do you think will happen academic instruction for infants with out solutions for themselves. Even the next?,” “does that remind you of what such indirect guidance. youngest middle-class children prac- we did yesterday?” Middle-class par- Yet such instruction is quite differ- tice traits that make academic success ents are more likely to read aloud, to ent from what policymakers nowadays more likely when they negotiate what have fun, to start conversations, as an consider “academic” for young chil- to wear or to eat. When middle-class entree to the world outside. Their chil- dren: explicit training in letter and parents give orders, they are more dren learn that reading is enjoyable and number recognition, letter-sound cor- likely to explain why the rules are rea- are more motivated to read in school. respondence, and so on. Such drill in sonable. There are stark class differences not basic skills can be helpful but is un- But parents whose jobs entail fol- only in how parents read but in how likely to close the social class gap in lowing orders or doing routine tasks they converse. Explaining events in learning. exude a lesser sense of efficacy. Their the broader world to children, in din- Soon after middle-class children children are less likely to be encour- ner talk, for example, may have as become verbal, parents typically draw aged to negotiate clothing or food. much of an influence on test scores as them into adult conversations so chil- Lower-class parents are more likely to early reading itself. Through such con- dren can practice expressing their own instruct children by giving directions versations, children develop vocabu- without extended discussion. Follow- laries and become familiar with con- Middle-class children’s ing orders, after all, is how they them- texts for reading in school. Educated selves behave at work. So their chil- parents are more likely to engage in self-assurance is dren are also more likely to be fatalis- such talk and to begin it with infants enhanced in after-school tic about obstacles they face, in and and toddlers, conducting pretend con- activities that some- out of school. versations long before infants can un- times require large fees Middle-class children’s self-assur- derstand the language. Typically, for enrollment and al- ance is enhanced in after-school activi- middle-class parents “ask” infants most always require ties that sometimes require large fees about their needs, then provide answers for enrollment and almost always re- for the children (“Are you ready for a parents to have enough quire parents to have enough free time nap, now? Yes, you are, aren’t you?”). free time and resources and resources to provide transportation. Instructions are more likely to be given to provide transporta- Organized sports, music, drama and indirectly (“You don’t want to make tion. dance programs build self-confidence so much noise, do you?”). Such in- (with both trophies and admiring adult struction is really an invitation for a spectators) and discipline in middle- child to work through the reasoning opinions. Lower-class children are class children. Lower-class parents more likely to be expected to be seen find the fees for such activities more and not heard. Inclusion this early in daunting, and transportation may also adult conversations develops a sense be more of a problem. In many cases, Civil Rights of entitlement in middle-class chil- such organized athletic and artistic ac- Movement Book dren; they feel comfortable address- tivities are not available anywhere in ing adults as equals and without def- lower-class neighborhoods. So lower- Our new book, Putting the erence. Children who want reasons class children’s sports are more infor- Movement Back Into Civil Rights rather than being willing to accept as- mal and less confidence-building, with Teaching (produced with Teach- sertions on adult authority develop in- less opportunity to learn teamwork and ing for Change), provides exten- tellectual skills upon which later aca- self-discipline. For children with sive resources to help teach about demic success in school will rely. Cer- greater self-confidence, unfamiliar the Civil Rights Movement in a tainly, some lower-class children have school challenges can be exciting; such way that highlights the important such skills and some middle-class chil- children, who are more likely to be contributions of rank-and-file par- dren lack them. But, on average, a from middle-class homes, are more ticipants in the Movement and sense of entitlement is social class- likely to succeed than those who are that connects students with based. less self-confident. present-day social movements. A Parents whose professional occupa- Homework exacerbates academic valuable resources for teachers, tions entail authority and responsibil- differences between middle- and community organizers, etc. ity typically believe more strongly that working-class children because For further information and to they can affect their environments and middle-class parents are more likely order a copy, go to www. solve problems. At work, they explore to assist with homework. Yet home- civilrightsteaching. org. alternatives and negotiate compro- work would increase the achievement mises. They naturally express these gap even if all parents were able to

8 • Poverty & Race • Vol. 13, No. 5 • September/October 2004 assist. Parents from different social middle-class professional parents tend to lower-class children, but such les- classes supervise homework differ- to associate with, and be friends with, sons compete with children’s own self- ently. Consistent with overall patterns similarly educated professionals. images, formed early in life and rein- of language use, middle-class parents Working-class parents have fewer pro- forced daily at home. — particularly those whose own occu- fessional friends. If parents and their pational habits require problem solv- friends perform jobs requiring little ing — are more likely to assist by pos- academic skill, their children’s images Culture and Expectations ing questions that decompose problems of their own futures are influenced. and that help children figure out cor- On average, these children must Partly, there may be a black com- rect answers. Lower-class parents are struggle harder to motivate themselves munity culture of underachievement more likely to guide children with di- to achieve than children who assume that helps to explain why even middle- rect instructions. Children from both that, as in their parents’ social circle, class black children often don’t do as strata may go to school with completed the only roles are doctor, lawyer, well in school as white children from homework, but middle-class children teacher, social worker, manager, ad- seemingly similar socioeconomic gain more in intellectual power from backgrounds. Middle-class black stu- the exercise than do lower-class chil- Teachers and counselors dents don’t study as hard as white dren. middle-class students, and blacks are Twenty years ago, Betty Hart and can stress doing well in more disruptive in class than whites Todd Risley, researchers from the school to lower-class from similar income strata. This cul- University of Kansas, visited families children, but such ture of underachievement is easier to from different social classes to moni- lessons compete with understand than to cure. Throughout tor the conversations between parents children’s own self- American history, many black students and toddlers. Hart and Risley found images, formed early in who excelled in school were not re- that, on average, professional parents warded in the labor market for that spoke over 2,000 words per hour to life and reinforced daily effort. Many black college graduates their children, working-class parents at home. could only find work as servants, as spoke about 1,300, and welfare moth- Pullman car porters or, in white-col- ers spoke about 600. So, by age three, lar fields, as assistants to less quali- children of professionals had vocabu- ministrator or businessperson. fied whites. Many Americans believe laries that were nearly 50% greater Even disadvantaged children now that these practices have disappeared than those of working-class children usually say they plan to attend college. and that blacks and whites with simi- and twice as large as those of welfare College has become such a broad rhe- lar test scores now have similar earn- children. torical goal that black eighth graders ings and occupational status. But la- Deficits like these cannot be made tell surveyors they expect to earn col- bor market discrimination, even for up by schools alone, no matter how lege degrees as often as white eighth blacks whose test scores are compa- high the teachers’ expectations. For all graders respond in this way. But de- rable to whites, continues to play an children to achieve the same goals, the spite these intentions to pursue educa- important role. Especially for black less advantaged would have to enter tion, fewer black than white eighth males with high school educations, dis- school with verbal fluency similar to graders actually graduate from high crimination continues to be a big fac- the fluency of middle-class children. school four years later, fewer eventu- tor. The Kansas researchers also tracked ally enroll in college the year after Evidence for this comes from the how often parents verbally encouraged high school graduation, and even continued success of employment dis- children’s behavior, and how often fewer persist to get bachelor’s degrees. crimination cases — for example, a parents reprimanded their children. A bigger reason than affordability prominent 1996 case in which Texaco Toddlers of professionals got an aver- is that while disadvantaged students settled for a payment of $176 million age of six encouragements per repri- say they plan on college, they don’t to black employees after taped conver- mand. Working-class children had feel as much parental, community or sations of executives revealed perva- two. For welfare children, the ratio peer pressure to take the courses or to sive racist attitudes, presumably not was reversed, an average of one en- get the grades to qualify and to study restricted to executives of this corpo- couragement for two reprimands. hard to become more attractive to col- ration. Other evidence comes from Children whose initiative was encour- lege admission officers. Lower-class studies finding that black workers with aged from a very early age are prob- parents say they expect children to darker complexions have less labor ably more likely, on average, to take perform well, but are less likely to market success than those with lighter responsibility for their own learning. enforce these expectations, for ex- complexions but identical education, Social class differences in role mod- ample with rewards or punishments for age and criminal records. Still more eling also make an achievement gap report card grades. Teachers and coun- almost inevitable. Not surprisingly, selors can stress doing well in school (Please turn to page 10)

September/October 2004 • Poverty & Race • Vol.13, No. 5 • 9 (GAP: Continued from page 9) school should be redefined to cover narrow the achievement gap only by more of the early childhood, after- attempting to duplicate such experi- evidence comes from studies in which school and summer times when the ences. blacks and whites with similar qualifi- disparate influences of families and Provision of health care services to cations are sent to apply for job vacan- communities are most powerful. lower-class children and their families cies; the whites are typically more suc- Because the gap is already huge at is also required to narrow the achieve- cessful than the blacks. One recent age three, the most important new in- ment gap. Some health care services study trained young, well-groomed and vestment should probably be in early are relatively inexpensive, like school articulate black and white college childhood programs. Pre-kindergarten vision and dental clinics that cost less graduates to pose as high school gradu- classes for four-year-olds are needed, than schools typically spend on many ates with otherwise identical qualifi- but barely begin to address the prob- less effective reforms. A full array of lem. The quality of early childhood health services will cost more, but cations except that some reported con- programs is as important as the exist- likely can’t be avoided if there is a true victions for drug possession. When ence of programs themselves. Too intent to raise the achievement of these youths submitted applications for lower-class children. entry level jobs, the applications of The association of social Policies to make stable housing af- whites with criminal records got posi- fordable to low-income working fami- tive responses more often than the ap- and economic lies with children and policies to sup- plications of blacks with no criminal disadvantage with an port the earnings of such families records. achievement gap has should also be thought of as educa- So the expectation of black students long been well known tional policies — they can have a big that their academic efforts will be less to educators. impact on student achievement, irre- rewarded than efforts of their white spective of school quality. peers is rational for the majority of The association of social and eco- black students who do not expect to many low-income children are parked nomic disadvantage with an achieve- complete college. Some will reduce before television sets in low-quality ment gap has long been well known to their academic effort as a result. We daycare settings. To narrow the gap, educators. Most, however, have can say that they should not do so and, care for infants and toddlers should be avoided the obvious implication: To instead, should redouble their efforts provided by adults who can create the improve lower-class children’s learn- in response to the greater obstacles they kind of intellectual environment that ing, amelioration of the social and eco- face. But as long as racial discrimina- is typically experienced by middle- nomic conditions of their lives is also tion persists, the average achievement class infants and toddlers. This re- needed. Calling attention to this link of black students will be lower than quires professional care-givers and low is not to make excuses for poor school the average achievement of whites, child:adult ratios. performance. It is, rather, to be hon- simply because many blacks (espe- After-school and summer experi- est about the social support schools cially males) who see that academic ences for lower-class children, simi- require if they are to fulfill the public’s effort has less of a payoff will respond lar to programs middle-class children expectation that the achievement gap rationally by reducing their effort. take for granted, would also likely be disappear. Only if school improvement needed to narrow the gap. This does proceeds simultaneously with social and not mean remedial programs where economic reform can this expectation Helpful Policies lower-class children get added drill in be fulfilled. math and reading. Certainly, remed- If we properly identify the actual iation should be part of an adequate Richard Rothstein (rr2159@ social class characteristics that produce after-school and summer program, but columbia.edu) is a research associate differences in average achievement, only a part. The advantage that of the Economic Policy Institute, a we should be able to design policies middle-class children gain after school visiting professor at Teachers College, that narrow the achievement gap. Cer- and in summer likely comes from self- Columbia University, and the author tainly, improvement of instructional confidence they acquire and awareness of Class and Schools. Using Social, practices is among these, but alone, a they develop of the world outside, Economic, and Educational Reform to focus on school reform is bound to be from organized athletics, dance, Close the Black-White Achievement frustrating and ultimately unsuccess- drama, museum visits, recreational Gap (Teachers College Press, 2004). ful. To work, school improvement reading and other activities that de- This article is adapted from a summary must combine with policies that nar- velop inquisitiveness, creativity, self- of that book prepared for the October row the social and economic differ- discipline and organizational skills. 2004 issue of American School Board ences among children. Where these After-school and summer programs Journal. ❏ differences cannot easily be narrowed, can be expected to have a chance to

10 • Poverty & Race • Vol. 13, No. 5 • September/October 2004 Social Class, But What About the Schools? by Pedro A. Noguera

Long before publication of Social internet; and less likely to be in a school lar support for the idea of improving Class and Schools, I was a fan of Ri- that is safe and orderly. Rothstein does public education and using it as a ve- chard Rothstein’s work. As a New not argue that improving these condi- hicle to promote opportunity and so- York Times columnist for several tions would not help poor children; he cial mobility. Like Rothstein, I agree years, Rothstein’s commentaries on simply suggests that this is not where that schools cannot be expected to ad- education were distinguished by his the emphasis for change should be dress the effects of poverty on chil- ability to bring common-sense insights placed. He focuses instead on the fam- dren alone, but from a tactical stand- to complex policy issues. In a field ily background of poor children and point I believe it makes sense to sup- where policy typically is driven by the multi-faceted effects of poverty, port the idea of advancing equity by ideology and the latest reform fad, factors that clearly have an influence expanding educational opportunities, Rothstein’s perspectives were fre- on achievement but which are harder rather than dismissing such efforts as quently a breath of fresh air, and I to address. Rothstein argues that im- unrealistic or hopelessly unattainable. often found myself clipping the articles Put more simply, reducing poverty and to share with students and colleagues. improving schools should not be Hence, I was not surprised to find Reducing poverty and treated as competing goals. Both are myself in complete agreement with improving schools necessary, but for the time being at most of the arguments in his new book. should not be treated as least, there is far greater support for In fact, many of the points he raises competing goals, but improving education. about the ways in which poverty in- for the time being at There are other parts of Rothstein’s fluences the academic performance of least, there is far argument that I also take issue with: poor children, I have made myself (my his arguments regarding minority stu- 2003 book, City Schools and the greater support for dent attitudes toward school (I contend American Dream). Like Rothstein, I improving education. that oppositional attitudes are often have often taken issue with those (like produced in school); his narrow focus the Thernstroms and The Heritage on Black and white students at a time Foundation) who assert that there are proving school conditions would not when Latinos and Asians are the fast- “no excuses” for the achievement gap lead to elimination of the achievement est growing groups nationally; and his between Black and white, or middle- gap. While this may be true, I find it lack of attention to the difference that class and poor children. As Rothstein hard to understand how any reason- highly qualified teachers can make in makes clear, lack of health care, inad- able person could argue that improv- influencing student outcomes. equate nutrition or inability to secure ing the abysmal conditions present in But most of all I am troubled by stable housing has an effect on the so many schools serving poor children his dismissal of the high-performing/ achievement of poor students, and would not have a positive effect on high-poverty schools that have been those who claim that children whose learning outcomes. documented by The Education Trust basic needs have not been met should My other point of disagreement and others. While there may indeed do just as well as more privileged chil- with Rothstein concerns his argument be a bit of exaggeration about some of dren are either lying or delusional. that some of the money being spent to these schools, I know from my own Despite my concurrence with improve schools should be redirected research and experience (see my ar- Rothstein on number of educational to address issues such as health care ticle “Transforming High Schools” in issues, there are at least two disturb- and housing that contribute to the hard- the May 2004 issue of Educational ing aspects to his main argument that ships experienced by poor children in Leadership) that such schools do ex- I take issue with. First, there is sub- America. My disagreement on this ist, and while they may not close the stantial evidence that the schools poor point is political rather than substan- achievement gap as some have claimed, children attend are more likely to be tive. While I agree that much more they do succeed in reducing academic overcrowded, underfunded and staffed needs to be done to address the needs disparities. The existence of such by inexperienced teachers. Poor chil- of poor children in America, such as schools is the most important evidence dren of color are also more likely to providing access to quality early child- available that the quality of schools attend schools that are segregated by hood programs, I also know that there poor students attend does matter. I’m race and class; less likely to have ac- has not been much political will or not sure if Rothstein would argue cess to the rigorous math and science support for taking on these issues since against this point or why he does not courses needed for college; less likely the War on Poverty in the 1960s. weigh in more heavily on the need to to have access to computers and the There is, however, substantial popu- (Please turn to page 12)

September/October 2004 • Poverty & Race • Vol.13, No. 5 • 11 (NOGUERA: Continued from page 11) ultimately is where Rothstein and I good schools can make for students disagree, and while I strongly endorse who are lucky enough to get access to do more to improve schools. In all the attention he directs toward the ef- them. likelihood, it is because his goal is to fects of poverty on achievement, I call for greater attention to the effects believe that the book he’s written is Pedro A. Noguera ([email protected]) of poverty rather than seeing so much not really about schools, it’s about is a professor in the Steinhardt School emphasis placed on reforming schools. what he thinks schools cannot do. The of Education at New York University While I don’t have a problem with that limitations he identifies are certainly and the Director of the Center for Re- emphasis, I do think it is important to real and profoundly important, but search on Urban Schools and Global- show what effective schools can do to what he pays insufficient attention to ization. ❏ promote student achievement. This is the extraordinary difference that

Don’t Lose the Battle Trying to Fight the War by John H. Jackson

In the year that we commemorate Rothstein’s work, indicated that the Civil Rights Project and The Advance- the 50th anniversary of Brown v. Board lack of socioeconomic opportunity led ment Project indicates, African Ameri- of Education and the 40th anniversary to family instability in poor black com- can students are more often sent to the of passage of the 1964 Civil Rights munities and gave rise to a “culture of office for “subjective” offenses and are Act, Richard Rothstein’s “Even the poverty” which often leads to unfa- more often penalized for offenses their Best Schools Can’t Close the Race vorable sociological outcomes. white peers are not penalized for. In Achievement Gap” highlights the im- Thus, while Clark, Moynihan and this case, the bulk of the problem lies portance of our nation’s commitment now Rothstein provide an accurate di- less with the student’s actions than the to address people of color’s socio-eco- agnosis of the symptoms that lead to system of discipline which labels a nomic ills as a tool for addressing and the racial achievement gaps that we see similar act “disruptive” on one hand closing the racial achievement gap. in school systems across the nation, the and “acceptable” on the other. Here, In theory, I wholeheartedly support the answer lies in ensuring that teach- Rothstein’s assertion that it is not by The remedy is not as ers have the professional development accident or outrageous misfortunes that needed to understand and educate the many of the areas that have the lowest simple as Rothstein population that sits before them. Fur- achievement levels are urban areas indicates. thermore, removing students from this populated by poor people of color who “culture of poverty” won’t alone close are confronted with many social chal- the achievement gap, as numerous lenges—people who often also have the remedy is not as simple as Rothstein studies have proven that even minor- lowest opportunity levels. This has indicates. Rothstein’s approach seems ity students in wealthier areas, on av- been a challenge that has begged for to indicate that by wiping out the so- erage, have lower test scores than their an answer for over a century. cial challenges that exist in urban com- white peers in similar areas. Immediately following passage of munities the racial achievement gap While Rothstein’s approach to ad- the 1964 Civil Rights Act, in 1965, will also disappear. Its underlying tone dressing the problem identifies a sig- Dr. Kenneth Clark, noted expert so- suggests, that many of the educational nificant barrier in addressing the gap, cial scientist in the Brown case, de- barriers that produce the achievement it does not account for the gap, nor scribed in his classic text, Dark Ghetto: gaps are centered in the student’s so- should it absolve schools of their re- Dilemmas of Social Power, the psy- ciological background rather than in sponsibility to ensure that there are chology and pathology of urban life. the institutions that are charged with highly qualified teachers in the class- Like Rothstein’s, Dr. Clark’s analy- educating all students—regardless of rooms, appropriate class sizes and ad- sis highlighted the outcomes rooted in socioeconomic background. For ex- equate resources. historical and contemporary forms of ample, Rothstein asserts that African If history is to be our guide in ad- discrimination against populations who American students are “more disrup- dressing this challenge, in 1964, Presi- were blocked access to educational and tive” in class than their white peers. dent Lyndon B. Johnson launched a economic opportunities. That same His assertion is likely rooted in the fact national War on Poverty. One of the year, Senator Daniel Moynihan headed that these students are more often re- first steps he took to address it was up a commission which issued a re- ferred to the office for discipline and working to pass the Elementary and port, The Negro Family: The Case for penalized more than their white peers. Secondary Education Act, which out- National Action, that again, like However, as research by the Harvard lined the federal government’s role in

12 • Poverty & Race • Vol. 13, No. 5 • September/October 2004 ensuring equal educational opportuni- Simplistic and Condescending ties for all children—through teacher quality, resource equity (Title I) and by Jenice L. View other components. Today, 40 years later, Title I is yet to have been fully Pity the low-income person who, Blacks is indeed shrinking, and if funded, and in 2004 President Bush by virtue of lousy wages alone, is con- many of the current Black achievers and Congress failed to fully fund the sidered an incompetent parent. Let’s are first-generation middle-class, from reauthorization of the Act (The No patronize her who is unworthy of talk- where did they all come? How do we Child Left Behind Act)—falling more ing with or reading to her child or help- explain the circumstances of their birth than $8 billion short of the resources ing with homework (following a 16- and their low-income parents and the required to give states and districts hour shift or her second job) because differences in outcomes? In other what is needed so that teachers can she cannot be relied on to do it cor- words, how is it that being poor one teach and students can learn in all com- rectly. And, should the low-income generation ago was less of a barrier to munities. parent feel too fatigued or too defeated achievement than now? Perhaps it is Thus, it remains difficult to mea- by racism at the end of a hard day’s due to the worsening income and sure the true weight the “culture of pov- work, let’s nevertheless encourage his wealth gaps between rich and poor of erty” has on the racial achievement gap kids to address him “as an equal and all races and ethnicities, a fact that is in education when the first battle—ad- without deference” in order to pro- neither irrelevant nor in the control of dressing the “culture of ensuring edu- mote the same sense of entitlement that cational opportunities to some and de- middle-class kids feel and use to their How is it that being nying them to others”—has yet to have academic advantage. been won. Nonetheless, the strength poor one generation Simplistic and condescending? No ago was less of a of Rothstein’s current work is not in less than Rothstein’s article. So, to his diagnosis of the war on poverty that give Rothstein the benefit of the doubt, barrier to achievement stills needs to be fought, but the con- let’s first assume that the supporting than now? text that his work provides to energize evidence for some of the more outra- stakeholders to pick up arms to address geous claims about urban, low-income the battle that exists in their local African American families are con- parents or teachers. The final para- schools and districts—the battle to en- tained within the book’s endnotes, and graph of the article makes the most sure equal access to a high-quality edu- are more current than the 20-year-old sense: cation for all students. If we win data he cites on (rural? White?) Kan- The association of social and eco- enough of these battles, we will surely sas families. While he seems to have nomic disadvantage with an win the war on poverty. no direct experience with low-income achievement gap has long been well African American families, we can known to educators. Most, how- John H. Jackson (jjackson@ hope that the citations include infor- ever, have avoided the obvious im- naacpnet.org) is National Director of mation about the cultural supports and plication: To improve lower-class Education for the NAACP, Chairman transformations of the last 40 years in children’s learning; amelioration of of the new National Equity Center, and the wake of legal desegregation, in- the social and economic conditions Adjunct Professor of Race, Gender cluding those within the Black church. of their lives is also needed. Call- and Public Policy at the Georgetown Secondly, his international com- ing attention to this link is not to Univ. School of Public Policy. ❏ parison is not credible because the ar- make excuses for poor school per- ticle fails to address native language formance. It is, rather, to be hon- literacy of dark-skinned immigrants to est about the social support schools Request Europe and Japan compared with the require if they are to fulfill the native language literacy of white Eu- public’s expectation that the for Syllabi ropeans. In addition, it is not clear if achievement gap disappear. Only the data he cites on parental occupa- if school improvement proceeds si- We’ve received several syllabi tion and student literacy hold constant multaneously with social and eco- for courses dealing with race and for language proficiency. nomic reform can this expectation poverty issues. We’d like to list Thirdly, the impact on urban com- be fulfilled. them (and how to access them) in munities of the crack cocaine epidemic a future issue of P&R. If you cannot be overlooked, leaving behind Jenice L. View ([email protected]) is teach/taught or are taking/took children with impaired health and a middle school teacher at a public such a course, please pass on grandparents to compensate for the charter school in Washington, DC and (preferably by email) such a syl- failings of addicted parents. co-editor of Putting the Movement labus. Finally, if the wealth gap between Back into Civil Rights Teaching. ❏ middle-class whites and middle-class

September/October 2004 • Poverty & Race • Vol.13, No. 5 • 13 Inequality and the Schoolhouse by Stan Karp

Richard Rothstein asks how much NCLB’s absurd “adequate yearly which have no record of success as schools can be expected to overcome progress” formulas, is being used to school improvement strategies and the staggering inequality that contin- label public schools as failures, with- which promise to do for schooling what ues to define our society. It’s the right out providing the resources and strat- the not-so-free market has done for question. Educational inequality— egies needed to overcome them. To health care and housing. (Just how se- whose manifestations go well beyond expect schools to wipe out long-stand- rious this privatization agenda is and test score gaps—is perhaps the central ing academic achievement gaps while how cynically concern for achievement problem our schools face. How we denying them substantial new re- gaps is being manipulated to advance deal with it will go a long way toward sources and leaving many of the so- it is currently a major point of differ- determining whether our society’s fu- cial factors that contribute to this in- ence among those who otherwise share ture will be one of democratic prom- equality in place is not a formula for a common interest in addressing issues ise or growing division. of educational inequality.) Weighing the ability of schools to Fifty years after Brown v. Board of compensate for the inequality that ex- Schools need pressure Education schools are being rightfully ists all around them is a question of from inside and out to taken to task for failing to deliver on balance, and there are dangers to be make reducing its promises. But the bill for that fail- found on both sides of the equation. educational inequality a ure, as Rothstein’s book shows, needs There’s little doubt that schools could to be itemized to include the appalling do more to bridge gaps between stu- more visible and more gaps in income, health care, nutrition, dents whose affluence provides private urgent priority. family support, housing, school fund- tutors and summer camps and those ing and other factors that translate into whose poverty or language status adds inequality in classrooms. Yes, we need only extra burdens. They could use the providing better education to those to press our schools to do a better job. inadequate resources they receive more who need it most. Instead, it’s a strat- But until society as a whole picks up efficiently and equitably. They could egy for eroding the common ground the tab for the equality it so often in- provide more academic supports, that a universal system of public edu- vokes as a goal, we will all continue more engaging curriculum, and more cation needs to survive. to pay a heavy price. effective, high-quality instruction It’s one thing to document academic Stan Karp ([email protected]) is They could move beyond a superfi- achievement gaps. It’s quite another a high school teacher in Paterson, NJ cial multiculturalism that “celebrates to use those data, as NCLB and many and an editor of Rethinking Schools. diversity” toward a deeper anti-racist of its supporters do, to promote a pu- With Linda Christensen, he co-edited practice that helps uncover why some nitive program of test-driven sanc- Rethinking School Reform: Views differences translate into access to tions, privatization and market reforms from the Classroom. ❏ wealth and power, while others be- come a source of discrimination and injustice. Schools could also design Even the Best Schools better systems for encouraging multi- sided accountability and promoting Can’t Do It Alone democratic collaboration with parents and communities. To do any of this, by Wendy Puriefoy schools need pressure from inside and out to make reducing educational in- At a time when the No Child Left achievement. equality a more visible and more ur- Behind Act all but monopolizes the de- Unfortunately, Rothstein uses his gent priority. bate on school reform, Richard astute observations about the manifes- That said, it seems to me that Rothstein raises important points that tations of these disparities to suggest Rothstein is essentially correct when underscore the broader context of pub- that the causes of the achievement gap he argues that schools face unreason- lic education—a context that deserves are personal or cultural, rather than able expectations from those who de- to be taken seriously now more than deeply systemic. In its focus on the mand schoolhouse solutions to the po- ever. To be sure, schools will benefit victims of the system rather than the litical, economic and social inequality when policymakers and communities system itself, Rothstein’s scrutiny that we allow to persist. Currently, the pay attention to the role that race and smacks of the old “cultural depriva- achievement gap, narrowly defined by class disparities play in shaping the all- tion” accounts of unequal success rates, test scores and, more recently, by too-predictable patterns of academic the idea that we can somehow explain 14 • Poverty & Race • Vol. 13, No. 5 • September/October 2004 away the achievement gap by finding nities to a fate, instead of engaging fault with the lifestyles of those who them and others to take an active, par- CORRECTION end up on the wrong end of it. The ticipatory role in the function of local trouble with this line of thinking is schools. In other words, the problem In our July/August issue, “Some that it often discourages comprehen- is not, as Rothstein claims in his title, Lessons from Brown for the Fair sive, systemic reform in favor of “re- that “Even the Best Schools Can’t Housing Movement” mistakenly forming” those who would benefit Close the Race Achievement Gap.” described Martin Luther King’s from it. If we are serious about creat- The reality is that Even The Best Chicago housing march as having ing lasting and effective reforms, we Schools Can’t Do It Alone. occurred in early 1968; the actual must look for problems within schools, Public schools rely on public in- date was August 1966. not pathologize children and families. volvement. Nonprofit organizations Rothstein’s analysis represents a par- like local education funds play a vital ticular barrier to comprehensive re- role in fostering both awareness of, issue, and at their best, Richard form because it fails to rise above a and responsibility for, education issues Rothstein’s observations help us to do set of superficial choices, reinforcing at the local level. When we engage just that. But contrary to their author’s a rhetorical dichotomy that plays di- communities in generating assets and implications, the observations are rel- rectly into the hands of those for whom ideas for public education, we help evant to school reform not because they supporting public education is not a expose its limits, but because they ex- priority. The fallacy of this dichotomy pand its potential. Only when we fully becomes clear when we realize that We must look for recognize the relationship between solving social disparities and improv- problems within community health, economic vitality ing public education are not compet- schools, not pathologize and academic achievement can we ing aims, but two parts of the same children and families. work towards solutions equal to the large one. Suggesting that we can ei- complexities of the task. Such a com- ther reform schools or address inequali- mitment to a shared public education ties in health care, housing, wealth and dispel the myth that a scarcity of re- may well be the first step towards a parental attention presents us with a sources forces us to choose between coherent new vision ensuring that ev- set of false choices that all of us and preparing our children at home and in ery child can benefit from a quality underprivileged communities in par- our communities or educating them in education. ticular have a vested interest in recon- the classroom. ciling. The danger of ignoring school- Of course, this is not an easy pro- Wendy D. Puriefoy (WPuriefoy@ based variables in favor of child-based cess. The first step towards building PublicEducation.org) is Executive variables is that it can have the flavor broader support for public education Director of the Public Education Net- of resigning underprivileged commu- is seeing public education as a broader work. ❏

PRRAC Update School of Govt., MLK Jr. Visiting Fellow in Urban Studies & Planning at MIT (2002-2004), and Acting Asst. • We thank and say goodbye to our • We’re delighted that Bill Emerson Sec. for Policy Development & Re- two summer law student interns, Hunger Fellow Rebekah Park has search at HUD from 1998-1999; Gre- Nicole Devero (who returns to joined our regular staff as Research gory D. Squires, Chair of the Soci- Georgetown Law Ctr.) and Nisha Associate. ology Dept. at George Washington Agarwal (who returns to Harvard Law Univ. and author of recent and forth- School but will continue to assist from • Changes in PRRAC’s Social Sci- coming books on , sprawl, afar with several PRRAC projects, in- ence Advisory Board: Richard Berk organizing access to capital, and preda- cluding our Dec. 3-4 Housing Mobil- leaves us, and we thank him for his tory lending; John Goering, profes- ity Conf.) years of service. And we have added sor in the Baruch School of Public 5 new members: Margery Austin Affairs, CUNY and author/editor of • And we welcome our new part-time Turner, director of The Urban several books on housing segregation intern: Elizabeth Grote, a 2nd year law Institute’s Metropolitan Housing & and discrimination; and Camille student at George Washington Univ. Communities policy center and HUD Zubrinsky Charles, Asst. Prof. in the who spent 3 years in the Neighborhood Deputy Asst. Sec. for Research from Dept. of Sociology and Research Defender Service in Harlem after 1993-96; Xavier de Souza Briggs, Assoc. at the Center for Africana Stud- graduating from Yale. Assoc. Prof. At Harvard’s Kennedy ies, Univ. of Pennsylvania. Welcome!

September/October 2004 • Poverty & Race • Vol.13, No. 5 • 15 What Teachers Know by Mark Simon

For teachers, the most dishearten- here, but was an important point in dent achievement and expand student ing aspect of the Administration’s “No the book.) Most importantly, they potential. Child Left Behind” agenda is the dis- talked of the weight it lifted from their Rothstein makes clear (not in his honesty in the goals and supposed suc- shoulders, allowing them to celebrate summary here but in the last pages of cess stories. No responsible educator human-scale improvement rather than the book) that part of his intent is to disagrees with the stated purpose of perpetually feeling bad about their provide an antidote to the demoraliz- leaving no child behind and closing the work. ing atmosphere that is driving the most achievement gap, but we must begin It is surprising how little we know creative, accomplished teachers out of with the truth. about teaching practices that cause stu- teaching, particularly fleeing schools The myth perpetrated by conserva- dents to succeed, particularly in high- with high-poverty students. This is a tive education reformers is that we can poverty schools. Ironically, the hyped significant issue. The class/race dis- abandon the war on poverty while ex- parities represented by vastly differ- pecting the children of the poor to NCLB has provided ent teacher working and student learn- achieve middle-class success in school cover for growing social ing conditions have widened to crisis simply by “raising expectations.” proportions. Rather than dismissing NCLB has provided cover for grow- inequality, de-funding of the need to correct the unequal distri- ing social inequality, de-funding of the the public sector, a bution of teacher talent as “politically public sector, a privatization agenda privatization agenda and and financially fanciful,” as he does increasingly unjustified by any re- a blame game that in the book, Rothstein should have search, and a blame game that scape- scapegoats the teacher included it under “Helpful Policies.” goats the teacher work force. The lib- work force. In all other respects, Class and Schools eral-conservative compromise that cre- brings the realities of what teachers ated the NCLB act seems premised on instinctively know to the policy-mak- an assumption that teachers aren’t re- myth-making success stories promoted ing table, hopefully before it’s too ally trying. The most talented teach- by The Education Trust, Heritage late. ers particularly resent the message. Foundation and purveyors of 90-90- Rothstein’s book provides ammunition 90 schools (90% poverty, 90% minor- Mark Simon ([email protected]) is for teachers and principals to respond ity and 90% meeting high academic the Director of the MCEA-JHU Cen- to the hype. standards), by making it sound so easy, ter For Teacher Leadership at Johns I recently gathered a group of ac- have actually distracted educators from Hopkins University. Formerly he complished teachers to discuss Class recognizing the more nuanced suc- taught high school Social Studies in and Schools. They agreed that the cesses that need to be documented and Montgomery County, Maryland, and book helped them to articulate what replicated. Class and Schools should served for 12 years as the elected presi- teachers already know – that teaching allow us to more realistically analyze dent of NEA’s 3rd largest affiliate, the lower-class kids well is tougher than what teacher behaviors, beliefs and Montgomery County Education Asso- teaching middle-class kids. The book school practices actually improve stu- ciation. ❏ doesn’t lessen their commitment to closing the achievement gap. It did lead them to want to personally take Family and School Matter new steps – walking tours of their school community and other strategies by Krista Kafer to get to know their students and fami- lies better; political activism to fight Richard Rothstein is right. His born to married parents is less likely for expansion of Head Start and other new book Class and Schools under- to have developmental delays or be- pre-school programs that help prepare scores what researchers like James havioral problems, repeat grades or be students and families for school; and Coleman, Derek Neal and Christopher expelled. Parents who read regularly initiating school- and district-wide con- Jencks have been saying for decades: to their children will see them grow as versations to reconsider decisions Life outside of school is the greatest readers. It is equally true that conflict which had narrowed the focus of edu- predictor of success in school. and instability at home will seep into cation to what is tested – de-valuing It should come as no surprise that a child’s performance in the classroom. important non-cognitive aspects. (This adults’ decisions impact their Even, so, demography is not des- is not covered in Rothstein’s summary children’s academic progress. A child tiny, and Rothstein admits as much.

16 • Poverty & Race • Vol. 13, No. 5 • September/October 2004 However, he discounts the power of a Successful schools are not limited the space left by a deprived home life, good school to make a difference. He to the private sector. Educators are it can go a long way. Giving kids ac- attributes the success of high-poverty/ replicating public school models like cess to schools of excellence will make high-performing schools identified by KIPP Academies around the country a difference. The Education Trust, The Heritage because they raise achievement among Unfortunately, the author’s solution Foundation and others to selectivity or low-income students. Whether public — to enact a host of new Great Soci- statistical anomaly. He believes such or private, such effective schools have ety programs — is unlikely to make a models may serve a few but are not much in common. Led by strong prin- difference. After almost four decades the answer for most. of Head Start, welfare, and federal His pessimism, however, is un- academic and after school programs, founded. Research shows that the Unfortunately, the there is little to show for the effort. greatest in-school predictor of academic author’s solution — to The focus has been in the wrong success is the quality of teaching. What enact a host of new place. Since family is the greatest de- happens 33 hours a week, 180 days a Great Society programs terminant of academic success, fol- year matters. lowed by teaching quality, these The late James Coleman, ground- — is unlikely to make a should be the focus of change. Poli- breaking researcher on the primacy of difference. cies that encourage marriage, parental socioeconomic influence, also found responsibility and access to good that Catholic and other private schools cipals and talented teachers, these schools will narrow the gap between achieved greater academic results with schools create an environment focused poor students and their middle-class poor students than public schools serv- on learning and character develop- peers. A healthy family and a good ing their peers. Similarly, albeit more ment. They build a solid foundation school are what a child needs most. recently, Harvard University’s Paul in the basics before moving to higher- Peterson found poor black students level material. Faced with many chal- Krista Kafer (krista.kafer@ using vouchers to attend private lenges, they often use a longer school heritage.org) is Senior Policy Analyst schools outperformed their public day or school year to get the job done. for Education at the Heritage Foun- school counterparts. While a school can never fully fill dation. ❏

Schools Count by Dianne M. Piché and Tamar Ruth

In Class and Schools, Richard single most important and effective off the potential of schools, however, Rothstein suggests that school reform “equalizer” of opportunity in our so- we should redouble our efforts to en- will not produce results unless and until ciety. If there is one place progressives sure that all children have access to the entire liberal social and economic can and should put their energy and schools that work, including: quali- agenda is fully enacted. He has sum- see results, it is in improving public fied teachers; a safe and supportive marized a one-sided collection of schools, because despite the persistence learning environment; and, critically, unsurprising and not very new studies of race and sex discrimination in the instruction that is not dumbed down about the impact of poverty, discrimi- job market, education remains the most but rather matched with the same high nation and class-related child-rearing promising ticket into the middle class standards taught in the suburbs and practices on student outcomes. His for black and Latino children. For required now by growing numbers of purpose is clear: to make a case that example, in the years following en- states in order to graduate. If states schools cannot be expected to produce actment of the Civil Rights Act of and school districts are not willing or the dramatic improvements demanded 1964 and the inception of Head Start able to desegregate schools with high by increasing numbers of parents and and Title I programs in 1965, along concentrations of poverty (and the voters, and called for under the No with court-enforced desegregation, we Prospects study conducted for the Na- Child Left Behind Act, because there saw dramatic narrowing of the gap tional Assessment of Title I, as well is very little schools can do to miti- between African American and white as other credible research, has made it gate achievement gaps caused prima- children on the National Assessment clear that one of the worst educational rily by non-school factors. of Educational Progress. environments is high poverty concen- Rothstein is wrong about the poten- Certainly there are “non-school” tration in the classroom), they and the tial and power of schools, and here’s factors that are difficult or outside the federal government should provide ad- why: power of schools to overcome, as ditional, carefully targeted resources First, education continues to be the Rothstein describes. Rather than write (Please turn to page 18)

September/October 2004 • Poverty & Race • Vol.13, No. 5 • 17 (PICHÉ/RUTH: Continued from page 17) handful of poor children can “defy the than the city schools they otherwise odds.” Rather, her own experience would have attended. to such schools and their students to both as an “at-risk” child growing up, • In California, one of the most enable them to succeed, including: and now as an educator, speaks pow- underfunded states, with huge num- highly qualified teachers; extended erfully to the fact that students can bers of poor and immigrant students, time (e.g., high-quality summer and succeed if we believe in and support Education Trust West has identified in- after-school programs); additional them. creasing numbers of high-poverty highly-trained professionals (e.g., • In the larger community of east- schools making or exceeding state reading specialists, master teachers/ ern Montgomery County, MD, where achievement targets. The Citizens’ coaches); professional development in both authors live, the public schools Commission in Civil Rights met with reading and other core subjects that is are majority nonwhite and enroll large and interviewed educators at some of aligned with the state’s standards; and numbers of poor and immigrant chil- these schools (and many others across sufficient pay or other incentives for dren. Under the superintedent’s lead- the country) as part of our Title I moni- good teachers to remain in these ership, a program of sensible, coher- toring project and found some com- schools. While a certain amount of ent instructional programs and inter- mon themes: 1) an overarching belief racial and economic isolation in among staff that all children could suc- schools is outside the control of school ceed (and with it a refusal to make ex- officials (the result of entrenched resi- Rothstein’s contention cuses or to blame parents); 2) a re- dential segregation), school boards that most successful lentless focus on literacy and getting retain control over student assignment high-poverty or high- all students to read on grade level by and attendance policies and ought to minority schools are the third grade; 3) continuous exami- do all in their power to reduce pov- flukes, statistical nation and use of data, including peri- erty concentration in classrooms; mag- odic assessment in reading and math, net schools, controlled choice and com- outliers or selective to implement instructional improve- pliance with NCLB’s new transfer pro- academies is not ments and changes; 4) a strong princi- visions can all help reduce isolation supportable. pal and senior staff who respect teach- and improve learning outcomes. ers, encourage collaboration and cel- Second, Rothstein’s contention that ebrate success but who also communi- most successful high-poverty or high- ventions targeted to poor neighbor- cate and enforce high standards; and minority schools are flukes, statistical hoods and schools is dramatically clos- 5) a sense of connection to a larger outliers or selective academies is not ing the gap, erasing many of the pre- community (e.g., through parent in- supportable. Despite Rothstein’s ef- school literacy deficits Rothstein as- volvement, adult volunteers, business fort to deflate and discredit as many serts are responsible for the gap at the partners and support from clergy and success stories as he can, our own ex- get-go. In one Title I school where faith communities). Significantly, perience in teaching and advocacy is author Piché volunteered and sent her very few of these schools received any completely consistent with The Edu- own children, the system’s intensive “extras” above and beyond their regu- cation Trust reports on successful and balanced literacy initiative brought lar district allocations for staffing and schools and the belief that success is nearly all low-income and non-En- materials and their Title I grants. But possible in far more schools (http:// glish-proficient second graders to or what they did with their resources was www2.edtrust.org/edtrust/dtm/), and above level on the district’s early read- to use them in the smartest, most effi- for many more students, than currently ing assessment, including a number of cacious ways to improve achievement. reported. There are success stories on children who might otherwise have Finally, Rothstein fails to address an individual, school and community- been consigned to special education. how schools and school officials them- wide basis all across the country, and • In St. Louis, where author Piché selves are often responsible for per- we each have been fortunate to live, has represented schoolchildren in an petuating and exacerbating achieve- witness and celebrate success every- ongoing desegregation case, we have ment gaps. Many more kids would where we go. For example: seen results from strategies to improve succeed in school and huge parts of • Last year, every single one of achievement. Specifically, over the the gaps would be erased if adults in author Ruth’s students (all nonwhite, last two decades, thousands of poor charge of schools ended policies and most eligible for free or reduced price black children from St. Louis trans- practices we know are bad for kids, meals, and many new to learning En- ferred to majority white and middle- including the following: glish) met the school district’s bench- class suburban districts, where their 1. The persistent, widespread ineq- mark in reading, and most far ex- parents could not afford to live. Un- uitable distribution of education re- ceeded the standard. Her experience der this program, the largest public sources, including teachers. In many as a classroom teacher, and her prior school choice program in the nation, parts of the country, rich students get work with poor Latino toddlers, re- the students achieved graduation and more and poor students get less. We futes Rothstein’s notion that only a college-going rates enormously higher are dismayed that in an entire work on

18 • Poverty & Race • Vol. 13, No. 5 • September/October 2004 the achievement gap, Rothstein makes ous impact of within-school discrimi- care, housing and income security are light of perhaps the most consequen- nation and other demeaning conduct. all very important. We do not dis- tial maldistribution of resources, that Under the category of “bad adult be- agree with any of them. of good qualified teachers. He writes havior,” we include both overtly and But, as discussed above, we dis- off closing the well-documented covertly biased remarks and practices, agree completely with his thesis in teacher quality gap as “politically un- schools that are managed as if they Class and Schools that schools them- realistic” (p.132 of the book). But were prisons and not places of learn- selves can do little to close achieve- this “in-school” problem is one of the ing, and school environments that are ment gaps. largest contributing factors to the unwelcoming to both students and par- Not only is Rothstein’s thesis incor- achievement gap in the first place. An ents. We also include the persistent rect, it also provides ammunition to extensive and growing body of research overuse of suspension, expulsion and an entrenched, retrograde education by Richard Ingersoll (Univ. of Penn.), so-called “zero tolerance” policies establishment desperate to excuse Jennifer King Rice (Univ.of Md.), that, as applied, deny students an on- achievement gaps at a time when there William Sanders (Univ. of Tenn.) and going opportunity to learn and often is a growing public consensus that such others has established that teacher have an adverse and disproportionate gaps are neither inevitable nor mor- quality is the most signficant in-school impact on minority and male students. ally defensible. This “establishment” variable that influences student includes some (though by no means achievement. Rothstein fails to all) public officials, school adminis- 2. Tracking and academic-content address how schools trators and classroom teachers who are gaps. Rothstein acknowledges that poor challenged, and in some cases person- and minority students have the same and school officials ally threatened, by the gap-closing educational aspirations as middle-class themselves are often promises and requirements of the bi- and white students: to go to college responsible for partisan NCLB. It also includes many and make a good living. But with a perpetuating and middle- and upper-class parents and set of widespread practices that expand exacerbating voters who, historically, have been rather than close gaps, schools them- achievement gaps. reluctant to send their tax dollars to selves make attainment of these goals the other side of town to improve the virtually impossible for many students. schools of poor and nonwhite students. These practices include: a) tracking 4. Dishonest grading and promo- Ironically, 50 years after Brown v. poor and minority students into whole tion practices. While we do not favor Board of Education, those very pro- classes or, in the earlier grades, large-scale retention, we also know, visions in NCLB that call for racial groups, where expectations and stan- as reported by the National Assessment and economic justice in the provision dards are low and remain low through- of Title I, that in general students in of educational resources (including out students’ educational careers; b) poor urban schools receive “A’s” for high-quality teaching) are among the counseling and steering similarly situ- work that would only pass for a “C” most threatening to some otherwise ated minority students into less chal- in the better-off suburbs. Children do moderate to liberal constituencies, in- lenging and dumbed-down high school not ultimately benefit when they are cluding the nation’s largest teachers’ classes, while white students are en- promoted from grade to grade with- union. These NCLB provisions in- couraged to take honors, Advanced out having attained the grade-level mas- clude the requirement that states put Placement and other more rigorous tery of reading and math skills neces- all schools on a trajectory to ensure courses; c) in some schools, not re- sary to do core subject coursework (in- that all children, including poor and quiring, encouraging or even offering cluding comprehending more complex minority students, can read and do a full sequence of college-preparatory texts) in succeeding years. Interven- math at the state’s own levels of pro- classes; and d) the failure of states and tion and additional assistance should ficiency within 12 years (a timeline districts to ensure that the same courses be immediate and targeted to prevent decried as unrealistic by many in the (e.g., algebra, biology) in fact have an accumulation of deficits that ulti- education establishment, but way too the same or comparable rigor across mately will lead to dropouts or failure long for most parents whose children school class and race lines. to meet graduation standards. will have fallen far behind, or dropped 3. Bad adult behavior toward chil- To his credit, Rothstein does ac- out, by the time the deadlines roll dren and their parents. In many high- knowledge the persistence of and harm around). Less widely discussed (per- poverty communities, there are adults caused by segregation, and calls for haps because the Bush Administration working in and supervising schools school and wider community (e.g., has been complicit in state and local who are downright disrespectful of stu- residential) integration, proposals with disregard of these provisions) are ad- dents and their families. Rothstein ad- which we agree. In addition, his rec- ditional requirements in NCLB to re- dresses the impact of discrimination ommendations for addressing poverty direct resources to the schools with occurring off school premises, but through progressive policies in the ar- greatest needs, including closing the neglects to acknowledge the poison- eas of preschool and child care, health (Please turn to page 20)

September/October 2004 • Poverty & Race • Vol.13, No. 5 • 19 (PICHÉ/RUTH: Continued from page 19) Rothstein Responds well-documented “teacher quality gap” between rich and poor schools. Several commentators charge that Some of the commentators (Pedro Compliance with this provision (which I devote too much attention to social Noguera, for example) appreciate the was supported by a coalition of civil reform and not enough to school im- need for complementary work on both rights organizations but opposed by the provement as a strategy for equalizing socioeconomic reform and school im- teachers’ unions) could involve the re- outcomes between blacks and whites. provement, but think I have gotten the deployment of highly qualified teach- Getting the balance right is difficult, balance wrong. Perhaps so. But ers at well-off schools to those with but the biggest obstacle to doing so is clearly the emphasis in public policy high concentrations of poverty and/or an excessive emphasis on the role of today is so exclusively on schools that the provision of economic and other schools. Were the obstacles reversed, a correction is in order. If, in some incentives for good teachers to remain I would have written a different book. unimaginable (in today’s political en- in high-needs schools. My summary for P&R of Class and vironment) future it swings too far in Most of us who support a broad Schools insisted that both are needed: the direction of social and economic progressive economic and civil rights “Improvement of instructional prac- justice, my book may serve a less use- agenda know the playing field in and tices is among [policies to narrow the ful purpose. out of school very likely will not be achievement gap], but alone, a focus Other commentators, however, who leveled in our lifetimes, nor during the on school reform is bound to be frus- claim to have read both the summary school careers of millions of poor Af- trating and ultimately unsuccessful. To and the book, stubbornly misrepresent rican American and Latino children work, school improvement must com- the argument as “school reform will now in or about to enter the public bine with policies that narrow the so- not produce results unless and until the school system. But we refuse to give cial and economic differences between entire liberal social and economic up on a generation or even a classroom children. . . . Only if school improve- agenda is fully enacted” (Dianne M. of children, or to stop pushing law- ment proceeds simultaneously with Piché and Tamar Ruth). These com- makers, school administrators and social and economic reform can [the mentators go on to assert, with no evi- other educators to do their very best, gap be closed].” In Class and Schools, dence whatsoever, that “education con- even as the Right Wing pushes for I explain that I devoted this work pri- tinues to be the single most important more shredding of the safety net and marily to the social and economic and effective ‘equalizer’ of opportu- the Left backs off its commitment to causes of the achievement gap, not nity in our society.” Is educational im- enforcing racial equality in education. because school inadequacies are unim- provement more effective than full It is not only reasonable but also mor- portant, but because our public discus- employment, anti-discrimination poli- ally imperative that we expect all sion of school and socioeconomic ef- cies in housing and labor markets, pro- schools to do right by all students. fects is now so imbalanced: Volumes gressive taxation, adequate public are produced weekly on how schools health, and unionization? Perhaps so, Dianne M. Piché (Diannepiche@ should improve (and with many of but I’d like to know the basis for such cccr.org), a civil rights lawyer, is Ex- them I agree), leaving me little to add. a claim. Recent research on inter- ecutive Director of the Citizens’ Com- But silence on the complementary im- generational mobility suggests that we mission on Civil Rights, where she was portance of social and economic re- are less mobile than we thought and principal editor of CCCR’s 2004 pub- form is deafening. less mobile than other advanced coun- lication, Choosing Better Schools: A In neither my summary nor my tries — most of which pay more atten- Study of Student Transfers Under the book do I deny that schools like KIPP, tion to social and economic equality No Child Left Behind Act, and their or those cited by The Heritage Foun- than we do. The conundrum is that it dation or The Education Trust, are 2002 report, Rights at Risk: Equality is difficult to overcome class differ- better than most and succeed in nar- in an Age of Terrorism. She also ences using a tool — schools — whose rowing the achievement gap. What I outcomes are themselves heavily in- teaches education law and policy at do deny is the claim of some of their fluenced by social class. the Univ. of Maryland-College Park. fans that such schools can close the As to Piché and Ruth’s historical Tamar Ruth (Tamar_Ruth@fc. achievement gap without simultaneous illustration, their memories are short. mcps.K12.md.us) is an award-winning social and economic reform. Interest- They correctly note that “in the years elementary school teacher in Mont- ingly, leaders of these schools, when following enactment of the Civil gomery Cty., MD, a doctoral student pressed, almost never make such Rights Act of 1964 and the inception in education policy and leadership at claims. They realize, as many policy of Head Start and Title I programs in the Univ. of Maryland, and on the analysts do not, that their efforts alone 1965, along with court-enforced de- board of the Montgomery County Edu- can be only modestly successful if so- segregation, we saw dramatic narrow- cation Assn. (the NEA teachers cioeconomic deprivation remains un- ing of the gap between African Ameri- union). ❏ addressed. can and white children on the National

20 • Poverty & Race • Vol. 13, No. 5 • September/October 2004 Assessment of Educational Progress.” that schools are not unique in their in- the need for better and more equitable But they fail to note that these were equality. school funding. We also set schools also years in which Medicare and Med- For conservatives, the reason for an up for failure when we discuss closing icaid were enacted, in which the mini- emphasis on schools is obvious. the achievement gap with schools mum wage was higher (in real terms, Schools are tax-supported institutions, alone. Even if school improvement and relative to the median wage) than and an attack on the public sector is at were our exclusive concern, would we it is today, when affirmative action in the core of a conservative agenda. Pub- achieve it by establishing goals (clos- employment was aggressively pursued, lic sector employees (both administra- ing the gap) that can’t be achieved and when suburban housing was opened for tors [“bureaucrats”] and unionized that make no distinction between the first time to black families, and workers) are enemies conservatives progress and failure? when black family size decreased (giv- love to have. Proposals to narrow in- Second, I think Professor Noguera ing children more parental time and come inequality, or to intervene in the may not be making the best estimate attention). Did these play no role? private housing, employment or health of political practicality. We’ve not, Surely, school improvements such as sectors, are attacks on private inter- after all, been so successful to date in Title 1 were important, but the years ests at the core of the conservative improving schools to the point where when the gap on the National Assess- base. Far better to blame schools for they come anywhere close to generat- ment narrowed were those when school all our ills. ing equal outcomes for children from and socioeconomic policies to address But why do liberals join in this dis- different social classes. And reforms inequality were pursued simulta- tortion? Is it because an excessive fo- like universal health care, full employ- neously. In the 15 years from 1965 to cus on school reform brings the flat- ment policy, more progressive taxa- 1980, the poverty of black children tering support of conservative allies? tion, adequate housing (consider the declined by over a third (from 66% to Truthfully, I don’t know the answer. Section 8 program) are not wild pie- 42% of all black children). Subse- in-the-sky ideas but policies that are quently, black children’s poverty con- In the long run, very much part of a practical agenda, tinued to decline, but at a much slower effective public policy and very much needed. Certainly, the rate. The 1965 to 1980 period provides present administration has no interest no support for believing that school cannot proceed from a in them, but the prior administration improvement can close gaps without myth. made some progress in all of them, complementary progress in the social despite daunting political opposition. and economic conditions experienced If, by some chance, advocates of so- by poor and minority children. (Piché Pedro Noguera offers a possible cial and economic reform can win and Ruth cite Head Start in support of explanation. He agrees that both so- greater power in our political institu- their complaint about my thesis, but cioeconomic and educational policy tions, we can hope that they will not as my summary and book stress, I re- are necessary to enhance equality, but be hindered by arguments of liberals gard expansion of early childhood pro- thinks that school improvement is more that only schools can make a differ- grams as one of the most important politically practical: “for the time be- ence. initiatives we can take. Whether this ing at least, there is far greater sup- Finally, I am gratified by the reac- is considered an educational or social port for improving education.” He tion of Mark Simon’s teacher group reform is beside the point.) worries that, in the present political to my book. One reason I wrote it was I frequently encounter caricatures environment when public funds are that I have been troubled by the de- of my argument, such as that of Piché scarce, advocacy of social and eco- moralization I have encountered and Ruth, by liberals who retain, with nomic reform will undermine support among dedicated, highly skilled and the Bush Administration and other con- for school improvement, leaving funds indefatigable teachers in schools serv- servatives, a belief that the only im- for neither. ing disadvantaged children. They portant barrier to equality worth ad- In response, I urge him to consider know that they make a difference and dressing is schools’ “soft bigotry of two points. First, in the long run, ef- bitterly resent being labeled “failures” low expectations” and other failures, fective public policy cannot proceed and considered indistinguishable from such as inadequate financing, classes from a myth. Denying the obvious teachers who are far less qualified, that are too large, and teachers who importance of socioeconomic condi- only because their students don’t are too poorly trained. While these are tions in perpetuating inequality may, achieve at the same level as privileged certainly barriers, I wonder why there in the short run, build support for suburban children. If my book can is such resistance to acknowledging school improvement efforts, but these help, in a small way, make them feel that there are others outside of schools quickly degenerate into an excessive better about their selfless and unrec- and that these are also worthy of at- attack on schools, as in present fed- ognized dedication, it will have been tention. One need not let schools off eral policy with its exaggerated em- worth it for that reason alone. ❏ the hook and deny that our educational phasis on testing, basic skills and ac- system is unequal in order to contend countability, and its nonchalance about

September/October 2004 • Poverty & Race • Vol.13, No. 5 • 21 Resources

When ordering items from 822-4600, the Resources Section, margareth@globalrights. Please drop us a line letting us know how useful please note that most org, www.globalrights. our Resources Section is to you, as both a lister listings direct you to org [8976] and requester of items. We hear good things, but contact an organization only sporadically. Having a more complete sense other than PRRAC. Prices • “National Survey of of the effectiveness of this networking function will include the shipping/ Latinos: Politics & Civil help us greatly in foundation fundraising work handling (s/h) charge Participation,” by the (and is awfully good for our morale). Drop us a when this information is Pew Hispanic Ctr. (2004), short note, letting us know if it has been/is useful to provided to PRRAC. “No is available at www/ you (how many requests you get when you list an price listed” items often pewhispanic.org/ item, how many items you send away for, etc.) are free. page.jsp?page=Reports- Thank you. Reports%20Section When ordering items from [8980] PRRAC: SASE = self- DLF, 877/590-6492. June 2004), is available addressed stamped • “The Long Shadow [8979] (possibly free) from envelope (37¢ unless of Jim Crow: Voter Manpower Demonstration otherwise indicated). Intimidation & Research Corp., 16 E. 34 Orders may not be placed Supression in America Poverty/ St., NYC, NY 10016- by telephone or fax. Today,” a joint report by 4326, 212/532-3200, Please indicate from the People for the Welfare www.mdrc.org [8938] which issue of P&R you American Way Fdn. and are ordering. NAACP, is available at • One Nation, Under- • “From Jobs to www.pfaw.org/pfaw/ privileged: Why Ameri- Careers: How California general/default. can Poverty Affects Us Community College aspx?old=16368 [8987] All, by Mark Rank (356 Credentials Pay Off for Race/Racism pp., 2004), has been Welfare Recipients,” by • Equal Justice Society published by Oxford Anita Mathur, Judy • Standing in the e-Newletter just began Univ. Press. [8930] Reichle, Julie Strawn & Shadows: Understanding publishing with its Chuck Wiseley (2004), is & Overcoming Depression Summer 2004 issue. To • “Financial Education available from the Ctr. in Black Men, by John get on (free) sub list, & Asset-Building Pro- for Law & Social Policy Head (2004), has been contact kkamisugi@ grams for Welfare (headed by former published by Broadway equaljusticesociety.org Recipients & Low- PRRAC Bd. member Alan Books. [8957] [8988] Income Workers: The Houseman) at Illinois Experience,” by www.clasp.org/Pubs/ • “Affirmative Action • Small Grants - Race/ Dory Rand, is a 15-page Pubs_PostsecEd [8959] & Diversity: The Begin- Ethnicity, Immigration article in the May-June ning of the End? Or the & Poverty: The Natl. 2004 Clearinghouse • The ASPIRE Act End of the Beginning?,” Poverty Ctr. seeks Review: Journal of (America Saving for by Mark R. Killenbeck proposals to broaden Poverty Law & Policy, Personal Investment, (32 pp., March 2004), is understanding of the available from the Retirement & Education) available ($10, relationships between Sargent Shriver Natl. Ctr. was introduced July 22, downloadable free) from race, ethnicity, immigra- on Poverty Law, 50 E. 2004, in both Senate (by the Educational Testing tion & poverty. Up to 5 Washington St., #500, Sens. Jon Corzine & Rick Service, Rosedale Rd., proposals will be funded, Chicago, IL 60602, 312/ Santorum) and & House Princeton, NJ 08541, with a max. $20,000 per 263-3830, admin@ (by Reps. Harold Ford, 609/921-9000, award. Drafts of funded povertylaw.org, Tom Petri, Pat Kennedy www.ets.org [8975] research will be presented www.povertylaw.org & Phil English). It would at a Jan. 2006 Ann Arbor [8935] establish a KIDS Account • “Next Steps for US conf. Feb. 15, 2005 for every newborn child Activists: Building on deadline, npcinfo@ • “Welfare Reform in in America. Inf. from Commitments Made at umich.edu [8993] Miami: Implementation, [email protected] the UN World Conf. Effects & Experiences of [8989] Against Racism” (57 pp. • “Exploring the Poor Families & Neigh- + Apps., March 2004) is Complexity of Diversity: borhoods,” by Thomas • “Hidden in Plain available (likely free) Culture, Competences & Brock, Isaac Kwakye, Signt: A Look at the from Global Rights, 1200 Ethics,” the 5th annual Judy C. Polyne, Lashawn $335 Billion Federal 18th St. NW, #602, DLF practitioners conf., Richburg-Hayes, David Asset-Building Budget” Wash., DC 20036, 202/ will be held Oct. 21-23, Seith, Alex Stepick & (16 pp. Summary, Spring 2004 in Miami. Inf. from Carol D. Stepick (41 pp., 2004) is available

22 • Poverty & Race • Vol. 13, No. 5 • September/October 2004 (possibly free) from the Criminal Community Develop- • “The State of the Corp. for Enterprise ment: A highly recom- South, 2004: Fifty Years Development, 777 N. Justice mended website is comm- After Brown v. Board of Capitol St. NE, #800, org.utoledo.edu/ Education” (78 pp., May Wash., DC 20002, 202/ • “Schools and Pris- index.html# [8984] 2004) is available ($20) 408-9788. The full study, ons: Fifty Years after from MDC, PO Box containing complete data Brown v. Board of • “States of Change: 17268, Chapel Hill, NC sources & methodology, Education” is a 6-page, Innovative Policy & 27516-7268, 919/968- is downloadable at 2004 report from The Investments for Stronger 4531, www.mdcinc.org www.cfed.org. [8990] Sentencing Proj. Contact Communities,” the [8937] Marc Mauer at the Proj., Fannie Mae Annual • “Why Did the 514 10th St. NW, Wash., Housing Conf., will be • “Affirmative Student Welfare Caseload DC 20004, 202/628- held Nov. 10, 2004 in Development: Closing the Decline?,” by Caroline 0871, mauer@ DC. Inf. from www. Achievement Gap by Danielson & Jacob Alex sentencingproject.org for fanniemaefoundation.org Developing Human Klerman, a 2004 report a copy. [8953] [8942] Capital,” by Edmund W. from the National Poverty Gordon (11 pp., Spring Ctr., draws on monthly • “Reforming the • NeighborWorks 2005 2004), is available caseload data from Oct. Police: Racial Differ- Training Institutes will (possibly free) from 1989 through June 2003. ences in Public Support be held in Denver (Feb. Educational Testing Contact: npcinfo@umich. for Change,” by Ronald 7-11), Mpls. (April 18- Service, Rosedale Rd., edu [9000] Weitzer & Steven A. 22), Boston (June 20-24), 19-R, Princeton, NJ Tuch, a 26-page article, DC (Aug. 22-26) & SF 08541-0001, 609/734- • “2004 Report on appeared in the May (Dec. 5-9). Inf. from the 5694, [email protected], Illinois Poverty: An 2004 issue of Criminol- Neighborhood Reinvest- www.ets/org/research/pic Analysis of Rural ogy. Reprints may be ment Corp., 800/438- [8948] Poverty” (40 pp.) is available from Prof. 5547, [email protected], available (possibly free) Weitzer, weitzer@gwu. www.nw.org/training • “Learn. Vote. Act. from the Illinois Poverty edu [8974] [8954] The Public’s Responsibil- Summit, c/o Heartland ity for Public Education” Alliance, 208 S. LaSalle • “Reforming Correc- (16 pp., 2004), a national St., #1818, Chicago, IL tions” is a 350-page, Education survey of public opinion, 60604, 773/728-5960, 2004 report on is available (likely free) x274, arynell@ California’s prison • “Using Rigorous from The Public Educa- heartlandalliance.org, system, which it charac- Evidence to Improve tion Network, 601 13th www.heartlandalliance.org terizes as “dysfunctional” Policy & Practice” is the St. NW, #900N, Wash., [9004] in recommending whole- 102-page report of the DC 20005, 202/628- sale reforms.The report Jan. 2004 High School 7460, www. emanates from a 40- Reform Conf. in New PublicEducation.org Community member panel appointed Orleans. Available (likely [8955] by Gov. Schwarzenegger free) from Manpower Organizing and chaired by ex-Gov. Demonstration Research • Natl. Ctr. for Rural George Deukmejian. Corp., 16 E. 34 St., Early Childhood Learn- • The Midwest Acad- Available at www.report. NYC, NY 10016-4326, ing website has been emy Organizers/Leaders cpr.ca.gov/corr/index.htm 212/532-3200, established at Mississippi Training Sessions will [8997] www.mdrc.org [8931] State Univ., www.ruralec. take place in Chicago msstate.edu [8961] (Oct. 18-22, 2004), • “Beyond the Maryland (Nov. 15-19, Economic/ Emperor’s New Clothes: • “No Child Left 2004), Chicago again The Role of the Central Behind: What’s in it for (March 7-11, 2005), Community Office in Systemwide Parents” is a traveling Calif. (June/July tba), Development Instructional Improve- informational and Chicago again (Oct. 17- ment,” by Larry Leverett, motivational workshop, 21, 2005), and Maryland • The Center for is the 10-page Summer organized by the Center again (Nov. 14-18, 2005). Neighborhood Technol- 2004 issue of Benchmark, for Parent Leadership. Inf. from the Academy, ogy 2001-2002 Annual the quarterly newsletter For inf. and scheduling, 28 E. Jackson Blvd., Report is available (likely of the Natl. Clearing- contact Kim Gardner, #605, Chicago, IL 60604, free) from the Ctr., 2125 house for Comprehensive 859/233-9849, x229, 312/427-2304, North Ave., Chicago, IL School Reform, 2121 K [8963] [email protected], 60647-5415, 773/278- St. NW, #250, Wash., DC www.midwestacademy.com 4800, www.cnt.org 20037-1801, 877/766- • Community Action [8965] [8951] 4277, AskNCCSR@ for School Reform, by goodschools.gwu.edu, Howell Baum (298 pp., www.goodschools.gwu.edu 2003, $22.95), has been [8936] published by SUNY

September/October 2004 • Poverty & Race • Vol.13, No. 5 • 23 Press, www.sunypress. • “Parent Leaders: An from the Ctr. for Law & edu/details.asp?id= untapped source in Families/ Social Policy (headed by 60769%20 [8968] education,” sponsored by Women/ former PRRAC Bd. the Ctr. for Parent Children member Alan Houseman), • Spencer Fdn. Disser- Leadership, will be held is available at tation Fellowships for in Kansas City, MO (Oct. www.clasp.org/DMS/ Research Related to 12-13, 2004) & Charles- • “Grandma and Documents/ Education have a Nov. ton, SC (Dec. 8-9, 2004). Grandpa Taking Care of 1093288195.25/ 10, 2004 application Inf. from 859/233-9849, the Kids: Patterns of marr_brf_5.pdf [8966] (which must be submitted x229, kgardner@ Involvement,” by Lina electronically) deadline. prichardcommittee.org, Guzman (7 pp., July • “High-Wire Act: Inf. from fellows@ www.CenterforParent 2004), is available Balancing Families & spencer.org [8969] Leadership.org [8983] (possibly free) from Child Jobs at Precarious Trends, 4301 Conn. Ave. Points,” by Jodie Levin- • “The Talent Develop- NW, #100, Wash., DC Epstein (2004), from the ment High School Model: Employment/ 20008, 202/572-6000, Ctr. on Law & Social Context, Components & www/childtrends.org Policy (headed by former Initial Impacts on Ninth- Jobs Policy [8932] PRRAC Bd. member Alan Grade Students’ Engage- Houseman), is available ment & Performance,” • “Where the Funds • The Evaluation at www.clasp.org/DMS/ by James J. Kemple & Are: Potential Use of Exchange devotes its 31- Documents/ Corinne M. Herlihy (47 Child Support Funds for page Summer 2004 issue 1092834134.29/ pp., June 2004), is Transitional Jobs to “Early Childhood High_Wire.pdf. [8967] available (possibly free) Programs,” by Abbey Programs & Evaluation.” from Manpower Demon- Frank, is a Aug. 2004 Issue (and subs) free from • “Religion & Mar- stration Research Corp., Policy Brief from the Ctr. the Harvard Family riage in Urban 16 E. 34 St., NYC, NY for Law & Social Policy Research Proj., 3 Garden America,” a 3-page, June 10016-4326, 202/532- (headed by former St., Cambridge, MA 2004 Research Brief, is 3200, www.mdrc.org PRRAC Bd. member Alan 02138, 617/496-4304, available (likely free) [8973] Houseman), available at hfrp_pubs@gse. from the Ctr. for Re- www.clasp.org/Pubs/ harvard.edu [8947] search on Child • “Equal from the Pubs_Job [8998] Wellbeing, Wallace Hall, Start: Promoting Educa- • Asian America & 2nd flr., Princeton Univ., tional Opportunity for Same-Sex Marriage, a Princeton, NJ 08544, all Preschool Children — Environment forthcoming (Fall 2005) [email protected]. Learning from the issue of Amerasia Jour- The Brief was adapted French Experience,” • “Environment nal, seeks submissions from “Then Comes from the French-Ameri- Threatened” (July 2004), (March 15, 2005 dead- Marriage?: Religion, can Foundation, is from the National line; Oct. 1, 2004 Race & Marriage in downloadable at Priorities Proj., offers deadline for 1-page Urban America,” by W. www.frenchamerican. state-by-state numbers on abstract or prospectus). Bradford Wilcox & org/htm/ Bush Adm. budget cuts to Submission guidelines at Nicholas Wolfinger, LeGoneFAFBrochure. clean water programs, the www.sscnet.ucla.edu/aasc/ downloadable (go to qxp.pdf [8986] quality of the environ- rdp2/index/html [8949] Fragile Families link, ment & a variety of click on Publications, • America’s Untapped health risks. Includes for “Moving Forward: Head then click on Working Resource: Low-Income each state: % of people Start Children, Families Paper Series at Students in Higher breathing unhealthy air; & Programs in 2003,” http:crcw.princeton.edu Education, ed. Richard number of people drink- by Katherine Hart & [8977] D. Kahlenberg (197 pp., ing unsafe water; % of Rachel Schumacher, a 2004), has been published people living in an area 2004 Policy Brief from • “Family Support by The Century Fdn., with 100 times the cancer the Ctr. for Law & Social during the Transition to www.tcf.org [9003] risk goal set by the Clean Policy (headed by former Adulthood” (4 pp., Aug. Air Act; and more. PRRAC Bd. member Alan 2004) is available (likely • “Preparing Today’s Available at Houseman), is available free) from the Natl. Leaders for Tomorrow’s nationalpriorities.org/ at www.clasp.org/Pubs/ Poverty Ctr., 1015 E. High Schools,” sponsored issues/env/threatened/ Pubs_New [8960] Huron St., Ann Arbor, by the Alliance for index/html?env1 [8991] MI 48104, 734/615-5312, Excellent Education • “I Can’t Give You [email protected], (1201 Conn. Ave. NW, Anything But Love: www.npc.umich.edu #901, Wash., DC 20036), Would Poor Couples [8985] will be held Oct. 3-5, with Children Be Better 2004 in DC. Inf. from Off Economically If • “Kids Count Puerto 703/739-4480, [8939] They Married?,” by Rico Project” (July Paula Roberts (2004), 2004), from the Natl.

24 • Poverty & Race • Vol. 13, No. 5 • September/October 2004 Council of La Raza, is the Housing [8934] first-ever comprehensive www.nhc.org/nhcimages/ • “The Legal Effect of data book on children in Paycheck_Counties2004.pdf Brown v. Board of • “HOPE VI Reloca- [8981] Puerto Rico. Contact tion: Moving to New Education on Public Sonia Perez, 787/641- Neighborhoods & Housing 50 Years 0546, [email protected] • “Private Sector Later,” the Housing & Building New Ties,” by Partnerships: Investing [8992] Susan Clampit-Lundquist, Development Law Inst. in Housing & Neighbor- 21st annual fall legal is a 33-page article in hood Revitalization,” by Vol. 15, No. 2 (2004) of conf., will be held Oct. Health Leigh Bezezekoff, Louis 12, 2004 in Baltimore. Housing Policy Debate. A. Galuppo, H. Beth Issue (and subs) free from Inf. from HDLI, 630 Eye • “Literacy and Health Marcus, Barbara St. NW, Wash., DC Fannie Mae Fdn., 4000 McCormick, Raymond in America” (48 pp., Wisconsin Ave. NW, No. 20001-3736, 202/289- April 2004) is available Schmidt, Robin 3400, Among the speak- Tower #1, Wash., DC Synderman, Kari Stanley ($15, but downloadable 20016-2804, fmfpubs@ ers: PRRAC’s Florence free) from Educ. Testing & Eleanor White (June Roisman, Elizabeth fanniemaefoundation.org 2004), is available Service, Rosedale Rd., [8944] Julian, Philip Tegeler and Princeton, NJ 08541, (possibly free) from the David Freund, as well as Natl. Housing Coal., 609/921-9000, • “The HOPE VI Georgetown Law Prof. www.ets.org/research 1801 M St. NW, #M-100, Sheryll Cashin. [8972] Program: What About Wash., DC 20006-1301, [8941] the Residents?,” by Susan 202/466-2121, • “NeighborWorks Popkin, Diane Levy, www.nhc.org/ • A New Online Laura E. Harris, Jennifer Training Inst.” will be Tutorial on the role of PrivateSectorFinal04.pdf held Dec. 13-17, 2004 in T. Comey, Mary [8982] race & ethnicity in the Cunningham & Larry New Orleans. Inf. from US healthcare system is Buron, is a 29-page Neighborhoood Reinvest- available from the Kaiser • “Boom or Bust? ment Corp., 1325 G St. article in Vol. 15, No. 2 Public Investment in Family Fdn. at (2004) of Housing Policy NW, #800, Wash., DC www.kaiseredu.org/ Homeownership” was a 20005, 800/438-5547, Debate. Issue (and subs) 2004 program hosted by Tutorials/index.cfm free from Fannie Mae www.nw.org/training [8964] The Population Resource [8940] Fdn., 4000 Wisconsin Ctr. Speakers included Ave. NW, No. Tower #1, • “Primary Care representatives of The Wash., DC 20016-2804, Census Bureau, Congres- Physicians Who Treat fmfpubs@ Miscellaneous Blacks and Whites,” by sional committees and the fanniemaefoundation.org Harvard Jt. Ctr. for P.B. Bach, H.H. Pham, D. [8945] • You Call This a Schrag, R.C. Tate & J.L. Housing Studies. Copies Democracy? Who of the speakers’ materials Hargraves, appeared in • “Housing Impact Benefits, Who Pays and the Aug. 5, 2004 New are available at Who Really Decides?, by Assessments: Opening www.prcdc.org/programs/ England Journal of New Doors for State Paul Kivel (218 pp., Medicine. The 15-page housing04/ $17.95), will be pub- Housing Regulation housing04.html [8999] article reports results of a While Localism Per- lished Oct. 2004 by Apex telephone survey of over Press, 800/316-2739. sists,” by Tim Iglesias, is • “Hedging His Bets: 4,000 doctors and an 82-page article [8943] analysis of over 150,000 Why Nixon Killed HUD’s appearing in the Summer Desegregation Efforts,” doctor visits by Medi- 2003 issue of Oregon Law • “Leading by Ex- care-covered Blacks and by Chris Bonastia, is a ample: Diversity, Review. Reprints may be 33-page article in the Whites. Among the available from Prof. Inclusion & Equity in conclusions: inferior Spring 2004 issue of Community Founda- Iglesias, Univ. San Social Science History. qualifications and less Francisco Law School, tions” (66 pp., 2004) is access to resources among Reprints may be available available (along with a 2130 Fulton St., SF, CA from the author, doctors who treat Blacks 94117-1080, 415/422- Resource Toolkit CD may contribute to racial cbonastia@ earthlink.net ROM) from California 5870, [email protected] [9002] disparities in the quality [8946] Tomorrow, 1904 Franklin of health care. [8995] St., #300, Oakland, CA • “The Housing Justice 94612, 510/496-0220; • “Paycheck to Pay- Network,” coordinated • The Calif. Primary check: Wages & Cost of may be free. [8950] Care Assn. 10th Anniv. by the Natl. Housing Law Housing in Counties, Proj., is holding its natl. Conf. will be held Oct. 7- 2004,” by Barbara • “Executive Excess 8, 2004 in Sacramento. meeting Oct. 3-4, 2004 in 2004: Campaign Contri- Lipman, is available DC, with a Basic Housing Inf. from the Assn., 1215 (possibly free) from the butions, Outsourcing, K St., #700, Sacto., CA Training Session Oct. 2. Unexpensed Stock Ctr. for Housing Policy, Inf. from the Law Project. 95814, 916/440-8170, 1801 K St. NW, #M-100, Options & Rising CEO x214, www.cpca.org 510/251-9400, x111, Pay,” from the Inst. for Wash., DC 20006-1301, [email protected] [8952] 202/466-2121, Policy Studies & United

September/October 2004 • Poverty & Race • Vol.13, No. 5 • 25 for a Fair Economy, is Housing Rights & lence is hiring a Director issue & supports other available at Evictions), 83 Rue de of Policy & Research & reforms that would www.faireconomy.org/ Montbrillant, 1292 a Director of Training & enhance political partici- press/2004/ Geneva, Switzerland, Technical Assistance and pation by people of color, EE2004_pr.html [8956] 41.22.7341028, Community Education & such as restoration of [email protected]. Development. Inf. from voting rights to former • “Litigating Eco- www.cohre.org [9005] Adelita Medina at the offenders.” Board is nomic, Social & Cultural Alliance, PO Box 672, looking to relocate the Rights: Achievements, Triborough Sta., NYC, project — DC, Atlanta Challenges & Strategies” Job NY 10035, EDAlianza@ and NC are leading (184 pp., 2003), featuring aol.com, www.dvalianza. contenders, but open to 21 case studies from Opportunities/ org [8970] other sites. Contact around the world (the US Fellowships/ Kristin Bradley-Bull, New • The Fannie Lou case study, by Maria Grants Perspectives Consulting Foscarinis and Andrew Hamer Project seeks an Gp., 1429 Broad St., Scherer, is “Using Civil Executive Director. The Durham, NC 27705, and Political Rights), is • The Natl. Latino Project is a natl. org. that [email protected]. [8971] available (no price listed) Alliance for the Elimina- “reframes campaign from COHRE (Ctr. on tion of Domestic Vio- finance as a civil rights

PRRAC'S SOCIAL SCIENCE ADVISORY BOARD

Frank Bonilla Paul Ong CUNY Department of Sociology UCLA School of Public Policy & Social Research Xavier de Souza Briggs Harvard Univ. Kennedy School of Government Gary Orfield Harvard Univ. Grad. School of Education Camille Zubrinsky Charles Department of Sociology, Univ. of Pennsylvania Gary Sandefur Univ. Wisconsin Inst. for Poverty Research John Goering Baruch College, City Univ. of New York Gregory D. Squires Department of Sociology Heidi Hartmann George Washington Univ. Inst. for Women’s Policy Research (Wash., DC) Margery Austin Turner William Kornblum The Urban Institute CUNY Center for Social Research Margaret Weir Harriette McAdoo Department of Political Science, Michigan State School of Human Ecology Univ. of California, Berkeley

Fernando Mendoza Department of Pediatrics, Stanford Univ.

26 • Poverty & Race • Vol. 13, No. 5 • September/October 2004 If You Are Not Already a P&R Subscriber, Please Use the Coupon Below.

❏ Sign Me Up! ❏ 1 year ($25) or ❏ 2 years ($45)

Please enclose check made out to PRRAC or a purchase order from your institution.

Name ______

Address ______

Address Line 2 ______

City, State, Zip ______

Telephone: ______email: ______

Mail to: Poverty & Race Research Action Council • 3000 Connecticut Avenue NW • Suite 200 Washington, DC 20008

September/October 2004 • Poverty & Race • Vol.13, No. 5 • 27 POVERTY & RACE RESEARCH ACTION COUNCIL Board of Directors

CHAIR TREASURER Don Nakanishi [Organizations listed for John Charles Boger Anthony Sarmiento University of California identification purposes only] University of North Senior Service America Los Angeles, CA Carolina School of Law Silver Spring, MD Florence Wagman Philip D. Tegeler Chapel Hill, NC Roisman President/Executive Director Sheila Crowley Indiana University VICE-CHAIR National Low Income School of Law Chester W. Hartman José Padilla Housing Coalition Indianapolis, IN Director of Research California Rural Legal Washington, DC Theodore M. Shaw Assistance Thomas Henderson NAACP Legal Defense Brenda Fleet San Francisco, CA Lawyers' Committee for & Educational Fund Office Manager Civil Rights Under Law New York, NY SECRETARY Washington, DC Cathi Tactaquin Rebekah Park john powell Elizabeth Julian National Network for Research Associate Kirwin Institute for the Dallas, TX Immigrant & Refugee Study of Race & Ethnicity S.M. Miller Rights Nisha Agarwal Ohio State University The Commonwealth Oakland, CA Elizabeth Grote Columbus,OH Institute William L. Taylor Law Student Interns Cambridge, MA Washington, DC

Poverty & Race Research Action Council Nonprofit 3000 Connecticut Ave. NW • Suite 200 U.S. Postage Washington, DC 20008 PAID Jefferson City, MO 202/387-9887 FAX: 202/387-0764 Permit No. 210 E-mail: [email protected] Website: www.prrac.org

Address Service Requested 9-10/04

Welcome new subscribers! Thanks to the Clearinghouse Review for helping to distribute Poverty & Race to its legal services subscribers. Clearinghouse Review is published bimonthly by the Sargent Shriver National Center on Poverty Law. With this issue, we also welcome the members of the Coalition for Human Needs, an alliance of national organizations working together to promote public policies that address the needs of low-income and other vulnerable populations.